Download   King’s College London School of Medicine  MBBS Year 4  Reproductive & Sexual Health Block 

Document related concepts

Menstruation wikipedia , lookup

Maternal health wikipedia , lookup

Women's medicine in antiquity wikipedia , lookup

Prenatal development wikipedia , lookup

Prenatal nutrition wikipedia , lookup

HIV and pregnancy wikipedia , lookup

Childbirth wikipedia , lookup

Prenatal testing wikipedia , lookup

Pregnancy wikipedia , lookup

Fetal origins hypothesis wikipedia , lookup

Obstetrics wikipedia , lookup

Maternal physiological changes in pregnancy wikipedia , lookup

Transcript
 King’s College London School of Medicine MBBS Year 4 Reproductive & Sexual Health Block Curriculum Guidelines CONTENTS SECTION 1 Introduction and general guidelines History taking in Obstetrics Obstetric physical examination History taking in Gynaecology Gynaecological diagrams: Female genitalia Pelvic examination SECTION 2 Seminar Programme Breast medicine seminars Genitourinary and psychosexual medicine Obstetric seminars Gynaecology seminars Rape and sexual assault Contraception Self directed learning advice (reading list) Symposia Guidelines SECTION 3 Skills assessment checklist Example questions 2 5 8 13 15 16 21 22 28 40 62 85 87 88 90 93 97 1 SECTION 1 THE PURPOSE OF THIS BOOK The aim of this book is twofold. Firstly, it will provide you with a guide to history taking and examination in obstetrics and gynaecology using methods that have been agreed across all RSH campuses. Secondly, it gives you detailed information concerning the form and content of the seminars that you will be given on your home campus and at your peripheral attachment during the Reproductive and Sexual Health rotation. The guidelines for these seminars should provide you with the requirements for the course and the subjects you can expect to be covered in examinations. We have also included learning objectives and guidelines for each symposium and the marking scheme that will be used by assessors when evaluating and signing up your skills during the block. Details of timetables, day to day rotas, course design, etc. that are specific to each site will be provided by your site leaders. HOW TO USE THIS BOOK Read the first sections on history, examination, presentation and general principles and have a general look over the seminar guidelines within the first two weeks of the block. This should give a you a good grounding in the principles and breadth of the subject early in the course. Read the seminar guidelines before each seminar and attempt the self‐assessment questions. We suggest you spend at least half an hour in preparation for each seminar. If you find a particular question difficult, do not be too concerned as the questions will be covered during the session. You will assimilate the information given to you in the seminars far better if you have spent some looking over its contents beforehand. Take the book to each seminar and refer to the relevant section during the session. Read the marking schemes for the skills assessment within the first two weeks of the block. These give you a very clear idea of what is expected when your essential skills are tested. You should aim to have a skill signed off as soon as you feel confident enough in that particular skill and close scrutiny of the marking scheme should speed up this process. OUTLINE OF THE COURSE The core teaching will be covered in tutorials, ward rounds, clinics, symposia and seminars. Symposia will be given on Wednesday mornings throughout the course. A programme of seminars in obstetrics, gynaecology and breast medicine will be delivered on each campus. 2 Student Selected Components (SSCs) in obstetrics and gynaecology take place on Fridays in Rotation 1. In rotations 2 and 3 you will be expected to take advantage of the clinical and teaching opportunities on a Friday and use your time efficiently and effectively to plan your elective (R2) and prepare for the exams (R3). GENERAL GUIDELINES The best way of learning obstetrics and gynaecology, genitourinary and breast medicine is by spending time with pregnant women and patients on the wards, in the clinics and doing acute on‐
call. This will be reflected in your clinical timetable. Senior colleagues who have experience and knowledge will be closely involved in both formal and informal teaching and you should seek their opinions as often as you can. Reading around the subject is vital. A suggested general reading list is included in this book and specific recommended texts are given for each seminar. Seminars and symposia are not substitutes for personal reading. Indeed, it is essential that you prepare for your seminars by reading about the topic and answering the self‐assessment questions before each seminar. You are expected to form your own opinions about current issues which have been mentioned in medical journals or even in the lay press and these subjects may be discussed at any time during the course. Our discipline is at the forefront of many moral and ethical issues in medicine, such as women’s rights in medical care, antenatal diagnosis, termination of pregnancy, HIV and assisted conception, amongst others. You are encouraged to explore these and discuss them, amongst yourselves and with members of staff. Several important areas of ethical discussion are indicated in the seminar programme. REQUIREMENTS OF THE CURRICULUM Our approach can be divided into the skills we expect you to attain, and the knowledge we wish you to acquire. The level of these should equip you for the start of postgraduate training in any speciality. 1. Skills By the end of the course we expect you to be able to take a comprehensive history and examine an obstetric or gynaecological patient, a patient with breast disease and a woman presenting at a genitourinary medicine clinic. By the end of the course you must demonstrate competence in vaginal examination including passage of a Cusco’s speculum and taking a cervical smear. These techniques will be explained and demonstrated to you via training models and the Gynaecology Teaching Associates (GTAs). The training models are available for practice in the clinical skills centre. When you perform a vaginal examination on an awake woman you must make sure that an appropriate chaperone is available (NOT a fellow medical student). Examples of appropriate people are a qualified doctor, midwife or nurse. One female should be present (other than the patient). A relative or friend of the patient is not acceptable. Note that written consent for a named student to perform a vaginal examination on a patient in theatre must be obtained prior 3 to her operation. Consent forms are available on the Virtual Campus and in DSU reception. A qualified doctor (ST4 and above) must co‐sign this consent form with you. Specific details of these and other skills that you must have signed up by the end of the course are given under section 3 in the course book. The sign‐ups can be done at either your base hospital or on your peripheral attachment. They must be done by consultant staff or specialist trainee (ST4 or higher), with the exception of normal labour which should be signed up by a senior midwife, and must be completed before you are allowed to take the end of block exam. 2. Knowledge The knowledge objectives are outlined in the core curriculum. More specific objectives are given in each seminar programme. You will be examined both at the end of the year on what is included in the symposia and seminar guidelines. 3. Work‐based assessments Workplace‐based assessment refers to the assessment of working practices based on what doctors actually do in the workplace, and is predominantly carried out in the workplace itself. Students should aim undertake 3 mini‐CEX assessments per rotation, approximately 1 every 4 weeks to reach a total of 9 for the year. Students should seek to undertake the assessments in a variety of clinical settings with a range of assessors (consultants, SpRs, senior nursing staff, senior midwives). Ideally each mini‐CEX should be followed by at least 10 minutes feedback, and the length of this feedback process should be recorded on the assessment form. Assessment forms are located in the student log book. 4 HISTORY TAKING AND EXAMINATION IN OBSTETRICS AND GYNAECOLOGY INTRODUCTION There are many ways to take a clinical history and perform an examination but a uniform approach at the start of training is desirable. With greater experience other methods may be tried but in the early stages of training this often leads to confusion. The details of history taking, examination and presentation explained below have been agreed across all sites at GKT and we would strongly advise you to keep to these guidelines. HISTORY TAKING IN OBSTETRICS Pregnant women often do not present with a disease but with a very significant biological event. Sensitivity and tactful questioning concerning this event are important. Most are young, fit women and the usual systematic enquiry learned for patients presenting with disease will generally be unrevealing. There is a move away from referring to pregnant women as ‘patients’ as the majority will not perceive themselves as such. The history concerning the present problem and the present pregnancy should be taken and presented first. The present pregnancy may be planned or unplanned, wanted or unwanted and the partner may or not remain involved. All these facts may have implications, especially for the provision of social and psychological support. There may have been a long period of infertility, or contraception may have failed. All these points should be elaborated when a careful history is taken in sequence from the preconception period but these questions should not be asked in an intrusive manner. The history of previous pregnancies may be complex but each one should be accounted for in chronological order. Particular tact must be exercised while eliciting a history of previous babies with complications, stillbirths and neonatal deaths. Such questioning must not be omitted but it is wise to wait until an appropriate time when trust has been established, in order to complete the history. Questions concerning social status and support, smoking, alcohol consumption and drug taking are those most commonly forgotten. Systematic questioning and logical follow‐up of positive responses are essential and a standard format of presentation is desirable. The following format is recommended. OBSTETRIC HISTORY PRESENTATION 


Name, age, occupation, gravidity/parity, how many weeks gestation. History of presenting problem in brief. Past reproductive history in chronological order: year, gestation, outcome, birthweight, condition at present, type of feeding – breast/bottle/mixed. 5 








History of present pregnancy. Presenting problem, presenting symptoms, investigations and management in detail. Gynaecological history, limited and relevant, cervical smear, contraception. Past medical and surgical history. Systems enquiry. Family history: heart disease, Thromboembolic disease, hypertension, diabetes. Social history: accommodation, family living at home, income, financial support. Drug history, allergies, smoking, alcohol consumption, diet. Summary of the above in two or three sentences and differential diagnosis if important to consider. HISTORY OF PRESENTING COMPLAINT This must be related to the pregnancy as it has progressed up to the present, culminating with the situation on the day of presentation. A history of vaginal bleeding, elevated blood pressure or proteinuria should be sought by direct questioning. Ask the woman what investigations have been done, what the doctors have said and what the plan of management is. The history must be summarised before an account of the examination is made. NB: Term is a period of time, not the estimated date of delivery (EDD) itself. Preterm is before 37 complete weeks of gestation. Post term is after 42 complete weeks of gestation. Preterm and premature are synonymous. 6 HISTORY OF PRESENT PREGNANCY Gravidity Number of pregnancies in total. Parity Number of pregnancies over 24 weeks (whatever the outcome). Estimated date of delivery (EDD) State whether determined by ultrasound scan (USS) or last menstrual period (LMP). Previous contraception, date of cessation, number of spontaneous periods prior to conception, regular/irregular periods prior to pregnancy. Any bleeding in early pregnancy. How many weeks of gestation now First trimester Pregnancy confirmed: when and how, symptoms of pregnancy, pregnancy test, ultrasound examination. Booking for antenatal care: what type midwifery, GP, shared or full hospital. Prenatal diagnosis (Down’s syndrome screening, chorion villous sampling). Second trimester Prenatal diagnosis (anomaly scanning, amniocentesis). When fetal movements felt by woman (quickening). Third trimester Findings at antenatal clinic. Whether fetal movements felt by patient. Test of fetal well‐being when indicated. Details of any hospital admission should be documented chronologically. 7 OBSTETRICS PHYSICAL EXAMINATION Ensure that an appropriate chaperone is present. The student and the expectant mother may both be nervous, but diffidence on the part of the student can lead to incomplete assimilation of the factors and erroneous findings. Basic skills in examination of the pregnant woman are essential and confidence will only be gained by appreciation of the method required and experience in applying it. General examination should be complete, but detailed systems examination such as neurological examination will only be done if appropriate. If a history has not been taken then the person examining the patient should introduce him or herself, explaining the purpose of the examination. Some introductory conversation will help all concerned to relax and the patient should be asked if she has any discomfort. Cold hands are unpleasant and may cause the patient to become tense. General comment: Body weight, height and fitness (“overweight for her height” or “high BMI” are polite ways of saying obese if you are presenting your findings in front of the woman). Hands: Radial pulse, palmar erythema. Skin: Signs of pregnancy – pigmentation; linea nigra, areola pigmentation of the nipples, freckles and facial pigmentation (chloasma gravidarum), spider naevi (like palmar erythema this is a normal change in pregnancy). Conjunctiva and mucous membranes: Pallor – anaemia. Thyroid: Sometimes normally slightly enlarged in pregnancy. Cardiovascular system: - Pulse, blood pressure. - Heart sounds and murmurs. - Many pregnant women have a third heart sound and systolic murmur. These reflect circulatory adjustment and the murmur, best heard at left sternal edge, comes from flow at the pulmonary valve. - Varicose veins Breasts: Changes of pregnancy – fullness, areolar pigmentation, Montgomery’s tubercles, condition of nipples, suitability for breastfeeding. Breast are not examined routinely any more in pregnancy. Students need to know of these changes but do not need to examine the breasts unless the woman comments on a problem. Oedema: Press examining finger on the skin over the tibia for ten seconds. Most pregnant women have some oedema, if it is sought; a major degree may be associated with pregnancy induced hypertension or pre‐eclampsia. Fundoscopy is relevant if the patient is hypertensive or diabetic. 8 Always be prepared to perform bedside urine testing with reagent strips. OBSTETRIC ABDOMINAL EXAMINATION This is a key element of examination of the obstetric patient. It must be comprehensive and presented in this order. Lie the woman in a semi‐recumbent position and ask if she has any discomfort; beware of supine hypotension. Inspection: Enlargement, shape. Pear shaped if singleton, globular if multiple. Pigmentation: Linea nigra (not always present), striae gravidarum. Surgical scars: especially transverse suprapubic hidden below the hairline and laparoscopy scares below umbilicus (it is often helpful to ask the woman if she has had any abdominal surgery as these scars are occasionally very difficult to see). Fetal movement: may be very obvious; if not, always ask the woman if there are fetal movements or any change in their pattern. Palpation: Comment on subcutaneous adipose tissue if relevant. Symphysis Fundal Height (SFH) in centimetres: Palpate from above downwards to locate fundus and mark this point. SFH is the distance to top (outermost point) of symphysis pubis allowing for subcutaneous fat. After 20 weeks the SFH approximates to the number of weeks. If reduced by 2cm before 36 weeks and 3cm thereafter, then intra‐uterine growth retardation is suspected and an ultrasound scan should be performed. Transverse lie is often associated with a reduced SFH. Lie and presentation: From 28 weeks onwards (and not usually before) in a woman who is not obese, palpation of the uterus will reveal the lie and presentation of the fetus. Prior to that the observation that fetal parts are felt is relevant but this may not occur until 26 weeks. Lie: Direction of the fetal spine with respect to the long axis of the uterus – longitudinal, oblique or transverse. Whilst the lie is being determined, consider fetal size and volume of amniotic fluid. Polyhydramnios (excess liquor) is indicated by a SFH greater than expected, difficulty in palpating fetal parts, and a tense cystic sensation. Oligohydramnios is indicated by ease in palpating fetal parts. Presentation: This is defined as the part of the fetus that is lowest in the uterus, usually just above or entering the pelvic inlet. If the lie is longitudinal, the presentation may be cephalic (head) or breech. With transverse or oblique lie, the presentation may be a shoulder or an arm. In most cases in later pregnancy the lie will be longitudinal and the presentation cephalic. If the head is normally flexed the lowest (presenting) part is the vertex which is the diamond‐shaped portion of the fetal head bounded by the anterior and posterior fontanelles and the parietal eminence. A malpresentation is any presentation other than the vertex. Extension of the head results in the fetal brow or face being the lowest part. Diagnosis of brow and face presentation is difficult though they usually present as a high head at term. The diagnosis can only be made with certainty by a combination of abdominal and vaginal examination in labour when the cervix is dilated and fetal 9 parts can be accurately identified per vaginum. If the presentation is uncertain it should be confirmed antenatally by ultrasound examination. Position: The denominator in cephalic presentation is the occiput and in breech presentation it is the sacrum. The relationship of the denominator to the inlet of the pelvis determines the position (right occipito‐anterior – ROA; occipito‐posterior – OP, etc.). Although it may be assumed that the back and occiput are aligned, position is not important antenatally except that identification of the back enables the fetal heart to be auscultated (see below); in labour, however, the position assumes greater importance and the findings at vaginal examination complement the abdominal findings. Station: Descent of the head is measured by the amount of the head palpable above the pelvic brim in fifths. Engagement is a precise observation of the point when the widest diameter of the fetal head passes through the pelvic brim. If it is engaged there is only two‐fifths, one‐fifth, or zero‐fifth palpable. Engagement is only applied to cephalic presentation. Attitude: This term means the relation of fetal parts to each other – the universally flexed fetal attitude. It is difficult to determine attitude on abdominal palpation but it may be abnormal when the head is deflexed or the extended legs of a breech are felt. Comments concerning fetal movements felt and uterine contractions palpated are also relevant. Auscultation: This is an essential skill which must be acquired by constant practice. It is most easily done with a Sonicaid Doppler probe and less easily with the fetal (Pinard) stethoscope. The position of the back and therefore the fetal thorax having been determined on palpation, the Sonicaid or Pinard is placed on the anterior abdominal wall facing this area. When using the Pinard the correct amount of pressure to indent the abdominal wall must be exerted and the hand must be removed from the stethoscope as this improves sound transmission. The fetal heart is quite low‐
pitched and the ear must be attuned to this. The normal fetal heart rate will be 110‐160 beats/minute. Fetal size is difficult to estimate clinically but the following may be kept in mind as a guide (means for an average European population): 22 weeks – 500 grams. 28 weeks – 1000 grams. 32 weeks – 1800 grams. 36 weeks – 2500 grams. 40 weeks – 3300 grams. Low birth weight is <2500 grams. Very low birth weight is <1500 grams. 10 PELVIC EXAMINATION IN OBSTETRICS Always perform an abdominal examination before a pelvic examination. During the antepartum period A speculum examination should be performed first in the following circumstances; a cervical smear is required, vaginal bleeding, rupture of membranes before labour. In pregnancy a bivalve speculum (Cuscoe’s) is appropriate. The purpose of digital examination is to examine the cervix, and assess uterine size in early pregnancy and gross pelvic normality. Around the time of labour Information about the cervix helps the management of labour. Firstly, it confirms that a woman is in labour if there is evidence of a change in the state of the cervix. Digital examination consists of:  Dilation – in centimetres.  Length – uneffaced is 3cm long, fully effaced is 0cm long.  Consistency – firm, soft or soft and stretchable.  Position – anterior, mid, sacral.  Station of presenting part – above, at or below ischial spines. An objective system of cervical scoring is particularly useful before induction of labour. The modified Bishop score (below) is one variation of a cervical score (formulated by Bishop, 1964). Score Points: Dilation of cervix (cm) Length of cervix (cm) Consistency (effacement) Direction Station of presenting part 0 0 3 Firm Sacral/Posterior Above ischial spines Score 0‐10 >6 Favourable (Ripe) <5 Unfavourable (Unripe) 11 1 1‐2 1‐2 Soft Mid At ischial spines
3 >3 0 Soft and stretchable Anterior VAGINAL EXAMINATION DURING ACTIVE LABOUR The emphasis of the findings relates to change: Dilation: 1‐10 centimetres. Effacement: Cervix usually effaced at this stage. Application of the cervix to the presenting part: Best assessed during a contraction. Station: With respect to ischial spines (+1,2,3 below or ‐1,2,3 above). Amniotic fluid: Absent, present, colour, smell. Caput formation: Swelling, oedema of scalp skin. Moulding: Position of sutures relative to each other. Grade 1 – touching but not overlapping. Grade 2 – overlapping but easily separated. Grade 3 – fixed overlapping. The association of failure of labour to progress (especially failure of the head to descend) and the appearance of a major degree of moulding is suggestive of mechanical obstruction in labour. 12 HISTORY TAKING IN GYNAECOLOGY Eliciting a gynaecological history and performing a gynaecological examination requires courtesy, thought and tact because of the personal nature of the subject. Basic skills must be developed and practised, recognising the specific nature of the problem. Privacy and ensuring modesty is maintained are important (ensure curtains are closed, blinds are shut and a sheet is given to patient to cover her up). The history should be taken in a relaxed and unhurried fashion. Rapport should be established at an early stage when an assessment of the patient’s age, social status and attitudes has been made. The interview should start with some introductory pleasantries and an account of the presenting complaint. Basic facts about this should be elicited but opportunities to obtain more detail will occur later. Menstrual history and reproductive history are essential in all cases. Further questioning should be structured logically around the presenting complaint, but account should be taken of the age, social status and attitudes of the patient. A sixteen year old who has never been sexually active with no supportive partner will require an entirely different approach than a married parous postmenopausal woman. A common fault is to fail to take an adequate sexual history in cases of subfertility, when timing, frequency and adequacy of intercourse should be elucidated. History of menstrual function starts at the menarche and finishes at the menopause. These are terms referring specifically to the menstrual changes, whilst puberty and the climacteric refer to the phases of life in which they occur. Reproductive history covers all pregnancies in chronological order, however long they lasted, irrespective of their conclusion. Previous gynaecological complaints, treatment and operations must be documented in detail. The elements of gynaecological history most commonly omitted inadvertently are time of the last menstrual period, presence of a vaginal discharge, pain or discomfort during coitus, contraception and when the last cervical smear was taken. 13 GYNAECOLOGICAL HISTORY PRESENTATION Although the history may have been elicited in one sequence a standard method of presentation is desirable as in obstetrics. This may seem rigid but it avoids the omission of important facts. 
Name, age and occupation. 
Parity/ Gravidity. 
Presenting complaint. 
History of presenting complaint. 
Menarche, menstrual history (last menstrual period, regularity and heaviness (menorrhagia) of menses, associated pain (dysmenorrhoea), relationship of complaint. Coitus, contraception and vaginal discharge. When was the last cervical smear taken? What was the result? Have there been smears in the past? +/‐ mammogram. IMB (Intermenstrual Abnormal Bleeding), PCB (Postcoital Bleeding), PMB (Post Menopausal Bleeding). 
Past gynaecological history. 
Past reproductive history: previous pregnancies in chronological order. 
Past sexual history: past or present sexual activity, pain on sexual intercourse (dyspareunia – superficial or deep). 
Past medical and surgical history. 
Systems enquiry. 
Family history: particularly VTE, CHD, osteoporosis, breast/ovarian cancer. 
Social history: accommodation, family at home, income, social and financial support. 
Drug history, allergies, smoking and alcohol consumption. 
Investigations and plan of management. Ask the patient what investigations have been done, whether she knows the diagnosis and what the management plan is. A short summary must be presented before examination. Too many negative elements of the history make it tedious, so select those that appear most relevant. ABDOMINAL EXAMINATION This must always be performed immediately prior to performing pelvic examination. This is partly because it helps to put the patient at her ease and partly because an abdomino‐pelvic mass may be anticipated before pelvic examination. Abdominal examination is performed according to the basic principles of inspection, palpation, percussion, and auscultation. 14 15 PELVIC EXAMINATION A chaperone is essential – a female nurse or doctor for male students. Inspection: Hair distribution. Labial appearance – colour, atrophy, discharge. Perineal scarring from episiotomy. Clitoral size and appearance. Descent of vaginal walls and cervix on straining. Leakage of urine on coughing (if bladder full) – stress incontinence. Palpation: Any external mass seen on inspection should be palpated and its size, shape, position and consistency noted. The Labia Minora should be separated using the non‐dominant hand to assess the size of the introitus to select speculum size. SPECULUM EXAMINATION: CUSCOE’S OR BIVALVE SPECULUM Know your glove size, a sizing chart is available in theatre. The method of use of the instrument should be practised using a simulator before the technique is used to examine patients. This avoids causing unnecessary discomfort to the patient. Always inform the patient fully of the procedure and treat her as gently as possible, being constantly mindful of her comfort. The majority of people perform this speculum examination with the patient in the dorsal position. If Uterovaginal prolapse is evident then examination with a Sim’s speculum in the lateral or Sim’s position becomes necessary. It is important to choose a medium Cuscoe’s speculum for a parous sexually active woman and a small Cuscoe’s speculum for a nulliparous sexually active woman. Do not perform speculum examinations in virgins and children in the outpatient clinic. When indicated examinations under anaesthetic are performed for removal of foreign bodies or to assess and treat any trauma in virgins and children. When the labia are widely separated, the anterior and posterior walls of the empty vagina are in apposition. The bivalve speculum should be inserted obliquely and then turned transversely to avoid traumatising the anterior area which is very sensitive. The speculum blades adequately lubricated should be passed downwards and posteriorly. Opening them should reveal the cervix but if not then gentle manipulation may be necessary. Good illumination is essential. Inspection: 




Discharge. Tone of vaginal walls. Cervix – erosions, polyps, lesions. Cervix – smear if indicated +/‐ swabs e.g. High Vaginal Swab, Endo‐cervical swab. Vaginal walls – assess descent on withdrawal of speculum. Do not close the valves of the speculum until you are sure the end of the speculum is beyond the cervix. If it is impossible to see the cervix then consider using a different speculum or using a Sim’s speculum if the problem is one of prolapse. How to take a smear: LBC (Liquid Based Cytology) – use a brush and ensure the entire surface of the cervix is sampled by placing the mid portion of the brush into the cervix and rotate clockwise twice. Then remove the brush and gently dab it into the liquid pot to ensure cells are placed into the liquid from the brush. Ensure the pot is labelled and the correct request form is completed. Cuscoe Bivalve Figure : Types of Speculum 17 Sim’s Speculum
BIMANUAL EXAMINATION This is usually performed after the speculum examination and the following should be noted: 




Consistency of cervix. Cervical os open or closed, cervical excitation. Uterus: o Size. o Consistency. o Anteverted/retroverted. o Tender/non‐tender. o Mobility to detect retroverted fixed uterus. Adnexae (the sides): o Masses palpable. o Tenderness. Pouch of Douglas (uterosacral ligaments): o Enlarged/scarred/nodularity. o Tenderness. Parting of labia, gel on glove, two fingers inserted posteriorly (to avoid pressure on more sensitive anterior structures). Insert fingers to posterior fornix. Rotate fingers anteriorly to feel cervix. Raise the cervix anteriorly and provide counter pressure with other hand on abdomen to feel uterus – size, mobility, texture. Then move fingers to each side to feel adnexae with counter pressure on abdomen to each side with other hand. Withdraw fingers from vagina.  Relevant objective terms are: o Firm = non‐pregnant uterus. o Soft = pregnant uterus. o Hard = fibroids or other pathology. o Bulky = slightly bigger than normal. A mass should be assessed in terms of:  Size.  Shape.  Consistency.  Whether it arises from the pelvis.  Whether it is mobile.  Whether it is tender. Remember bimanual examination requires both hands. The left hand is placed on the anterior abdominal wall and must be used to palpate deep into the pelvis against the counter pressure of the hand in the vagina. The vaginal hand is initially placed behind the cervix to assess the retroverted uterus, the Pouch of Douglas and the uterosacral ligaments. 18 Figure – bimanual examination How to perform a bimanual pelvic exam of a female (Video): Curated 6/13/08 Hosted by youtube.com. Creator’s Site www.wonderhowto.com/hot‐to‐perform‐bimanual‐pelvic‐exam‐female‐151888 SIM’S SPECULUM EXAMINATION Examination for UV prolapse should be performed with the patient in Sim’s position using a Sim’s speculum with a good light. Sim’s position is one in which the patient lies on the side with the knee and thigh drawn upward toward the chest. The descent of the cervix (uterine prolapse), anterior wall (cystocoele), posterior wall (rectocoele) or Pouch of Douglas (enterocoele) and the vaginal vault (post hysterectomy) may be assessed during maximum valsalva and gentle withdrawal of the instruments. The Sim’s speculum should be used to hold the posterior vaginal wall while a wooden spatula or cotton mounted on sponge holding forceps is used to assess the anterior vaginal wall and the cervix. Grade 1 = descent of any organ to the vaginal midplane. Grade 2 = descent to the hymenal ring. Grade 3 = descent to halfway through the introitus. Grade 4 = complete eversion. Subsequently the Sim’s speculum is placed to depress the anterior vaginal wall and the sponge folder or spatula is used to inspect the posterior wall for evidence of rectocoele, enterocoele, cervical or vault descent. 19 Figure : Mode of holding Sim’s Speculum 20 SECTION 2: SEMINAR PROGRAMME Breast medicine How to perform a breast examination BRE 01: Imaging, diagnosis and screening BRE 02: Breast cancer risk factors BRE 03: Management of breast cancer 22 24 25 26 Genitourinary and psychosexual medicine HIV infection: Clinical manifestations of early disease, principles for antiretroviral therapy Clinical manifestations of advanced disease Sexual history taking: Communication skills and genitourinary medicine/ The pre and post HIV test discussion Genital ulcer disease / Genital dermatoses Psychosexual medicine 35 37 38 Obstetrics OBS 01: Maternal physiology in pregnancy and labour OBS 02: Booking visit – normal antenatal care OBS 03: Pre‐pregnancy counselling OBS 04: Anaemia in pregnancy OBS 05: Anomaly scan and prenatal diagnosis OBS 06: Monitoring the fetus OBS 07: Management of labour/delivery OBS 08: Antepartum haemorrhage OBS 09: Preterm birth OBS 10: The third stage and puerperium OBS 11: Hypertension and proteinuria in pregnancy OBS 12: Medical disorders in pregnancy 40 42 43 45 48 50 52 54 56 58 59 60 Gynaecology GYN 01: Menstrual disorders GYN 02: Miscarriage and ectopic pregnancy GYN 03: Acute pelvic pain including cysts GYN 04: Dysmenorrhoea – chronic pain and endometriosis GYN 05: Amenorrhoea GYN 06: Pelvic mass GYN 07: Sub fertility GYN 08: Urinary problems GYN 09: Uterovaginal prolapse GYN 10: Menopause and osteoporosis GYN 11: Female genital mutilation GYN 12: Gynaecological oncology Rape and sexual assault Contraception: Community Reproductive & Sexual Health (CRaSH) 62 64 66 68 70 72 74 76 78 80 82 83 85 87 Self‐directed learning advice (reading list) 88 21 31 33 BREAST MEDICINE BREAST EXAMINATION 


You will need to examine the breasts and the regional lymph nodes. Examine the patient sitting up facing you and then lying on the couch‐ some changes are easier to see or feel in different positions. The principles of breast examination are the same as for any system – inspect and then palpate. Ask permission to examine breasts and offer a sheet to maintain modesty. 
Inspection Crouch to do this. You are looking for:  Size.  Symmetry.  Skin.  Puckering.  Peau d'orange.  Nodules.  Discoloration.  Ulceration.  Significant rash.  Inverted or asymmetrical nipples, signs of discharge from nipples. Ask the woman to lift her arms. If necessary, lift the breast to enable complete inspection. Again you are looking for the above. Palpation Tell the patient: ‘If you feel any discomfort during the examination, let me know.’ Breasts:  Start on the breast with no symptoms to ascertain how her normal breast feels. Palpate the breast, including areolae, with the finger pads. Do not use finger tips. Do the same with the symptomatic breast.  For nipple discharge, squeeze the nipple gently / press the areola to elicit this. Regional lymph nodes:  Supraclavicular and infraclavicular nodes Palpate above and below both clavicles using gentle but firm pressure.  Axillary lymph nodes You are feeling the tissue between the anterior axillary fold (pectoralis major) and the posterior axillary fold (latissimus dorsi). To do this, take the weight of a woman’s arm and feel behind the anterior axillary fold, pressing upwards, then towards the ribs with your 22 fingertips. This presses the axillary tissue against the underlying chest wall. Take the weight of the left arm with your left hand (palpate the axilla with your right fingers). For the right axilla take the weight of the right arm with your right hand (examine with axilla with your left fingers). Repeat the examination with the patient recumbent.  If a woman’s breasts are larger, you can roll her towards or away from you to ease examination of the outer quadrants and axillary tail. Wash your hands. Then leave the patient to get dressed in private. When you present your examination findings: If you see or feel any changes you must describe where these are.  Which quadrant of the breast (upper outer, upper inner, lower outer or lower inner). Use a clock‐face reference if the clinical sign is very close to the nipple‐areolar complex e.g. at the 2 o'clock position).  For a palpable lump state whether it is well‐defined or ill‐defined and give an approximate size in cm.  If you feel something behind the nipple / areolar complex this is ‘retroareolar’.  If a woman presents with breast pain and inspection with palpation are normal, state if there was discomfort on examination and which sites of the breast this was – is it generalized or localised to one place? The clinical examination is the first part of breast triple assessment i.e.  Breast clinical examination.  Breast imaging – mammogram and USS.  Breast biopsy (if appropriate). 23 BRE 01: IMAGING, DIAGNOSIS AND SCREENING Knowledge objectives By the end of a teaching session, the student should be able to:  Define the criteria for breast cancer screening by mammography.  Define the objectives of breast cancer screening.  Explain how the WHO criteria for screening apply to breast cancer.  Summarise the evidence for the effectiveness of screening.  Evaluate the advantages and disadvantages of breast cancer screening with mammography.  High risk screening with MRI, and advantages/disadvantages. Skills objectives The student should have observed and be able to describe the following procedures:  Image guided breast needle biopsy.  Needle localisation and surgical excision of non‐palpable breast lesions. Attitudinal objectives  The student should be able to explain the psychological morbidity which may be associated with screening for breast cancer.  Explain the ethical issues relevant to: o Treatment of cancers detected by screening. o ‘Over‐treatment’ or ‘under‐treatment’ of breast cancer. o Screening and assessment tests. Description of teaching session and methods to be used  Prior preparation by answering the self‐assessment questions.  Discussion of the questions.  Discussion groups; small groups reporting back to main group.  Lecture. Self‐assessment questions 1. What are the criteria for breast cancer screening by mammography? 2. What are the objectives of the breast cancer screening programme? 3. What are the stages of breast cancer screening? 4. What are the advantages and disadvantages of breast cancer screening? 5. What is the triple approach to assessment of abnormalities that are found on screening? Why is it used? 24 BRE 02: BREAST CANCER RISK FACTORS Knowledge objectives By the end of a teaching session, the student should be able to:  List the major known risk factors for breast cancer.  Explain the genetic inheritance of some breast cancers and describe the link between breast cancers and other cancers.  Describe the link between in situ and invasive breast cancer.  Understand the difference between stage and grade of breast cancer. Description of teaching session and methods to be used  Prior preparation by answering the self‐assessment questions.  Discussion of the questions. Self‐assessment questions 1. List the known risk factors for breast cancer and explain the relative risk of each of these in influencing the incidence of breast cancer in the general population. 2. How is the risk of some breast cancers influenced by genetic inheritance? 3. Describe the link between breast cancer and other cancers. 4. How does tumour grade, size and node status influence survival in patients with breast cancer? 25 BRE 03: MANAGEMENT OF BREAST CANCER Knowledge objectives By the end of a teaching session, the student should be able to:  List the prognostic factors used in determining treatment of breast cancer.  Analyse treatment options in terms of the breast and disease outside of the breast.  Describe the effects of risk factors for breast cancer on treatment outcome.  Describe the effects of tamoxifen and aromatase inhibitors on treatment outcome.  Define the following terms: o Local surgery. o Adjuvant therapy. o Primary chemotherapy. o Primary endocrine therapy.  List the common types of chemotherapy used in the treatment of breast cancer.  List common hormonal breast cancer treatments and their modes of action.  Describe at least one staging system.  Explain and contrast different methods of axillary node staging.  Describe the rationale behind adjuvant treatment and list the agents used. Skills objectives The student should be able to:  Interpret preoperative imaging, pathology and clinical notes to formulate a treatment plan.  Interpret the histological information obtained from surgery. Attitudinal objectives o Describe and illustrate how information is given to patients. o Explain the range of reactions that patients go through when undergoing treatment. o Describe role of the multidisciplinary team and the expected behaviour of members of that team. o Describe common expectations that women with breast cancer have. o Explain the ethical issues relevant to breast cancer treatment. Description of teaching session and methods to be used  Epidemiology.  20 minutes on surgery including management of breast/axillary reconstruction.  15 minutes on role of chemotherapy and endocrine management.  10 minutes on role of radiotherapy.  5 minutes on prognostic factors.  5‐minute review on current issues . 26 Self‐assessment questions 1. Discuss the information obtained from axillary surgery, including the impact on adjuvant treatments. 2. Describe the influence of oestrogen receptor status on choice of adjuvant systemic therapy. 3. Describe the benefits and disadvantages of sentinel node biopsy. 4. Describe common side effects of chemotherapy and how they are ameliorated. 5. Contrast and compare the range of side effects from breast cancer management. 27 SEXUAL HEALTH AND HIV UNDERGRADUATE SEMINAR TOPICS AND LEARNING OBJECTIVES Epidemiology of STIs and HIV  Understand the trends in STI rates in the UK.  Aware of the epidemiology of HIV.  Can identify the risk factors associated with STIs and HIV transmission.  Aware of the consequences of a late presentation with HIV. HIV pre‐test discussion (PTD)  Understand the importance of normalising HIV testing.  Identify the elements of PTD and what should be discussed.  Able to identify the key elements in giving an HIV result (negative and positive). Natural history and classification of HIV  HIV types, subtypes and strains.  Natural history of HIV.  Acquisition → AIDS (normal me periods).  Typical presentations at different stages. The diagnosis, management and prevention of opportunistic infections in HIV  Pneumocystis pneumonia.  Toxoplasma encephalitis.  Kaposi’s sarcoma.  Cryptococcal meningitis.  Mycobacterium tuberculosis.  Lymphoma.  Cryptosporidiosis.  Cytomegalovirus.  Oesophageal candidiasis.  Recurrent pneumonia.  Progressive multifocal leucoencephalopathy.  Plus several more. HIV therapeutics  Overview of HIV treatment principles.  Pharmacology of different classes of antiretroviral agents.  Appreciate significance and complexity of antiretroviral drug interactions.  Understand the role of post‐exposure prophylaxis (PEP) following sexual exposure in HIV prevention and when it should be prescribed. 28 Non‐medical needs of people with HIV  The variety of non‐medical needs, from the healthcare worker’s and patient’s points of view.  Why these needs are important.  What can be done by the multidisciplinary team to meet these needs. Viral hepatitis  Hepatitis A, B, C, D and E.  Basic virology.  Risk factors and transmission.  Tests and interpreting them.  Prevention and treatment of hepatitis in the GU/HIV setting. Sexual health services in context  History of services for STIs in the UK.  Systems in other countries.  Current national sexual health strategy.  Modernising the delivery of services.  Stigma and STIs / venereophobia.  Confidentiality, and threats to it.  Appropriate attitudes for healthcare workers.  Partner notification.  The issue of using a chaperone.  Surveillance in STIs.  GU medicine as a career choice. Recognition and management of male dysuria and urethral discharge  Recognise common causes of urethral discharge and dysuria.  Understand diagnostic techniques.  Come up with a management plan for a man presenting with urethritis. Management of vaginal discharge  Recognise the common causes.  Appropriate history/examination.  Appropriate investigations.  Appropriate treatment and follow‐up. Contraception  Overview of contraceptive methods: o Range of methods. o Principles of how they work. o Contraceptive choices and patterns of use.  Detailed information on individual methods to be covered in CRaSH session: 29 o Small group work. o Opportunity to handle methods. Genital ulcers and lumps  Different presentations of genital ulcers and lumps.  Diagnosis of ulcer/lump.  Treatment.  What to tell the patient. Management of pelvic pain/PID  Recognise the symptoms and signs of PID.  Appropriate differential diagnosis.  Appropriate investigations.  Appropriate management and follow‐up. Sexual assault and rape  Define rape.  Outline its prevalence.  Discuss the needs of complainants and the care they might require (such as emergency contraception, PEP and Hepatitis B vaccination). Ethical issues in sexual health  Recognise ethical issues in sexual health.  Appreciate the difference between ethical and legal obligations.  Understand the importance of confidentiality in sexual health. Sexual Health Adviser role  Understand who attends the sexual health service.  Recognises where health advisers fit into the sexual health service.  Aware of the role of partner notification in delivering public health.  Can explain how partner notification is achieved. 30 HIV INFECTION: CLINICAL MANIFESTATIONS OF EARLY DISEASE AND PRINCIPALS OF ANTIRETROVIRAL THERAPY Knowledge objectives By the end of the teaching session the student should be able to:  Define seroconversion.  Define HIV disease according to the CDC (Centres for Disease Control) criteria.  Define opportunistic infection.  Define primary prophylaxis.  Define secondary prophylaxis (or maintenance).  List the features of HIV seroconversion illness.  List the features of early HIV disease.  List the AIDS defining conditions. Skills objectives The student should be able to interpret:  CD4 count with regard for the normal range.  When to start HIV therapy.  HIV viral load to monitor effectiveness of treatment and adherence. Attitudinal objectives  The student should be able to explain and demonstrate the importance of all forms of communication both verbal and non‐verbal, being open and non‐judgmental. Description of teaching session and methods to be used  Prior preparation by answering the self‐assessment questions.  Discussion of the questions.  Interactive tutorial. 31 Self‐assessment questions 1. A 24‐year‐old man presented to A&E with a maculopapular rash, headache, sore throat and enlarged cervical LN’s. a. If these features are due to infection with HIV, by what name is the condition known? b. What is the main differential diagnosis? c. How would you determine if this is due to HIV disease by means of history and laboratory tests? 2. A 38‐year‐old patient from Africa has been controlled on antiretroviral therapy for 3 years, but recently his treatment had begun to fail. What are the possible reasons for this and how would you evaluate them? 3. Give the likely prognosis of the following 3 cases if they remain untreated, and explain your reasons. Explain how you would monitor them and whether you would offer them treatment, and if so, with what. a. A 24‐year‐old man with a CD4 count of 380cells/mm3 and viral load of 3,700copies/ml. b. A 58‐year‐old man, who had been infected for 1 year, with a viral load of 220,000 copies/ml and a CD4 count of 680cells/mm3. c. A 38‐year‐old African woman, who presented to the clinic well and with no previous HIV symptoms, with a viral load of 36,000copies/ml and a CD4 count of 40cells/mm3. 32 HIV INFECTION: CLINICAL MANIFESTATIONS OF ADVANCED DISEASE Knowledge objectives By the end of this teaching session each student should be able to:  Identify the main classes of drugs used to delay progression of HIV disease.  Describe the differential diagnosis of the following in a patient with HIV infection: o Rash. o Shortness of breath/cough. o Loss of consciousness/hemiparesis. o Meningism/headache. o Decline in cognitive function. o Visual disturbance. o Diarrhoea. o Wasting. o Jaundice.  Describe the indications for caring for an HIV infected person in isolation conditions on a ward (TB, C Diff). Self‐assessment questions 1. Construct lists of complications of HIV infection for each of the following systems: a. Pulmonary. b. Gastrointestinal. c. Neurological. 2. What are the commonest tumours associated with in HIV infection ? 3. List the manifestations of Pneumocystis carinii pneumonia. 4. What are the specific investigations and medications used in the management of PCP. 5. List the mycobacterial infections in HIV infection and explain the difficulty in diagnosing these infections. 6. List causes and management of diarrhoea in HIV infection. 7. List the fungal infections associated with HIV and the drugs used to treat them. 8. What are the causes of space occupying lesions in HIV? For each name investigations that may be used to diagnose them. 9. List the main classes of drugs used in the management of HIV. 10. A 32 year old HIV positive gay man presents with a 2 week history of shortness of breath? a. List the questions you would ask in relation to his presenting complaint. b. List 3 investigations to make a diagnosis. c. List the differential diagnosis and discuss the management of each condition. 11. A 42 year old HIV positive man with a CD4 count of 50cells/mm3 comes to you with a 1 week history of diarrhoea. List the possible causes of diarrhoea in an immunocompromised patient and describe the management of this patient. 12. A 29 year old woman with AIDS presents with severe headache and drowsiness. On examination she is febrile. She gives no history of meningism and on examination there are 33 no focal neurological signs. List the possible diagnosis and the urgent investigations required to safely manage this patient. Links with matrices 3 Acute and chronic breathlessness 5 Haemoptysis 6 Chronic cough 16 Dysphagia 17 Loss of appetite 19 Weight loss 23 Diarrhoea 61 Acute headache 62 Chronic headache 34 SEXUAL HISTORY TAKING (COMMUNICATION SKILLS AND GENITOURINARY MEDICINE / THE PRE AND POST HIV TEST DISCUSSION) Knowledge objectives By the end of this teaching session each student should be able to:  Describe the appropriate questions to elicit relevant information about a patient's sexual history.  List the barriers to obtaining a sexual history.  Describe relative risks of transmission of HIV for various sexual exposures.  Identify the key issues to be included in HIV pre‐test discussions.  Explain the reasons for undertaking pre‐test discussions. Attitudinal objectives By the end of a teaching session the student should be able to:  Describe how to create an atmosphere in which the patient feels able to explain & clarify a problem so that a diagnosis may be reached.  Explain the need to listen in a non‐judgmental way.  Describe his/her own prejudices regarding sexual health.  Explain the factors affecting sexual activity.  Describe the covert ways requests for help may be made.  Explain how doctors do not always have answers, and how they can share grief/pain without becoming emotionally involved.  Describe the effects the patient may have on the doctor.  Describe the vulnerability a patient may feel when having a genital examination.  Summarise the ethical issues relevant to obtaining a person's consent to an HIV test. Description of teaching session & methods to be used  Prior preparation by answering self‐assessment questions.  Discussion of the questions.  A tutorial presenting information concerning: o How to take a sexual history. o Reasons for HIV pre‐test discussions. o Checklist of topics to be discussed with patients prior to a test for HIV. o Assess a patient's risk of having acquired HIV.  Video on sexual history taking.  A group discussion about the use of language in genitourinary medicine consultations.  A participative exercise to identify barriers to discussing sex with patients, and consider how to overcome these. 35 Self‐assessment questions 1. Give an example of a question about partners that will convey a `closed attitude' to sexuality i.e. you are expecting an answer of monogamous heterosexuality. 2. Give an example of a question about partners that will convey an `open attitude' to sexuality. 3. List at least 4 sexual acts that it is important to include in a sexual history. 4. Explain why the question "Did you use a condom?" is a poor way to ask about barrier contraception. Describe a better way to elicit a truthful answer. 5. A 28 year old woman comes to the clinic worried she has "caught something down below" from someone she met on holiday. She is quite distressed. a. How will you begin the consultation? b. What words will you use to describe her genital anatomy? c. What questions will you ask her about symptoms and general health? d. Why is it important to ask where in the world sex took place, and where the contact was from? e. Give an example of a question which will determine the gender of the contact and convey an open attitude to sexuality. 6. A gay man who has had a number of HIV tests in the past 5 years requests another. He is reluctant to discuss why, and refuses to see a health adviser. a. What might you do to help to overcome his reluctance to discuss this? b. What are the reasons for a pre‐test discussion? c. How will you assess his risk of having contracted HIV? d. Describe how you would elicit information about: ‐sexual partner(s). ‐sexual acts. ‐condom use. 7. A 53 year old man presents with a contact slip and demands to know what infections he may have contracted. How do you respond to him? Links with matrices 56 Depression 57 Anxiety 36 GENITAL ULCER DISEASE/ GENITAL DERMATOSES Knowledge objectives By the end of the teaching session the students should be able to;  List the presenting features of genital herpes.  List the causes of genital ulcers due to sexually transmitted infection.  List the causes of genital ulcers due to other causes.  Describe the presenting symptoms and the clinical features of genital herpes.  List the principles involved in managing genital ulcers due to herpes simplex virus.  List the presenting features of primary syphilis and list the investigations performed to diagnose primary syphilis.  List two medications commonly used in the management of primary syphilis.  List the names of the tropical genital ulcer disease.  List the dermatological conditions presenting to a GUM Clinic.  List the typical features of lichen sclerosis affecting the genital area and its complications.  List the malignant lesions affecting the genital area. Description of teaching session & methods to be used  Prior preparation by answering self‐assessment questions.  Discussion of the questions. Self assessment questions 1. What are the common the presenting features of genital herpes? 2. List the causes of genital ulcers due to 1) sexually transmitted infection and 2) other causes. 3. Explain the principles involved in managing genital ulcers due to herpes simplex virus. 4. What are the presenting features of primary syphilis and the investigations performed to diagnose primary syphilis? 5. What two medications are commonly used in the management of primary syphilis? 6. List the names of the tropical genital ulcer disease. 7. Which dermatological conditions present to a GUM Clinic. 8. List the typical features of lichen sclerosis affecting the genital. What are its complications? 9. What malignant lesions can affect the genital area? Links with matrices 48 Chronic skin ulcers 105 Skin infections 106 Lumps in the skin 109 Vaginal discharge 122 Genital ulcers and warts 37 PSYCHOSEXUAL MEDICINE Knowledge objectives By the end of a teaching session the student should be able to:  Describe normal reproductive and sexual functioning.  Explain the range of sexual practice.  Explain the relevance of relationships in human reproduction and sexuality.  Describe how psychological disorders can present with sexual dysfunction.  Describe the impact of pregnancy on sexual functioning and relationships? Attitudinal objectives The student should be able to demonstrate:  An ability to understand a range of reproductive and sexual practice.  The ability to obtain an understanding approach in reproductive and sexual matters.  The student should be able to explain the ethical dilemmas regarding prescription of phosphodiesterase (5) inhibitors. Skills objectives The students must demonstrate efficiency in the following skills: • How to take a full history of human reproductive life cycle, and its impact on the individual. • How to take a full sexual history including the normal range of sexuality and its dysfunction. • How to take a full relationship history. • How to recognise psychological disorders e.g. depression, psychosis. • How to recognise psychological disorder associated with physical illness e.g. depression in malignant disease. • How to recognise psychological disorder in the absence of physical illness e.g. psychological causes for pelvic pain, dyspareunia. • Ability to counsel regarding risk factors in physical illness. • Ability to communicate difficult and sensitive information in the field of sexuality, malignancy or abnormal pregnancy. Description of teaching session and methods to be used • Prior preparation by answering the self‐assessment questions. • Discussion of the questions. Self assessment questions 1. Describe normal reproductive and sexual functioning. 2. Devise a classification for the range of sexual practice. 3. What is the relevance of relationships in human sexuality? 4. Which psychological disorders can present with sexual dysfunction? 5. What are the psychological problems associated with normal or abnormal sexual functioning? 6. What are the possible effects of psychotropic medication on sexual function? 38 7. What are the ethical dilemmas regarding prescription of phosphodiesterase (5) inhibitors? 8. What is the impact of pregnancy on sexual functioning and relationships? Links with matrices 56 Depression 57 Anxiety 39 OBSTETRICS OBS 01: MATERNAL PHYSIOLOGY IN PREGNANCY AND LABOUR Knowledge objectives By the end of a teaching session the student should be able to:  List the most important functions of the placenta.  List the most important effects in pregnancy of a) oestrogens and b) progesterone.  Describe the changes that occur in the uterine body and cervix from the first to third trimesters of pregnancy.  Explain the changes that occur in pregnancy for the following: o blood pressure, cardiac output, stroke volume, heart rate, peripheral resistance, uterine, skin and renal blood flow, oxygen consumption, tidal volume, residual volume, PC02, lower oesophageal sphincter tone, gastric motility, intestinal motility, renal blood flow, and ureteric muscle tone.  List the normal changes in the ECG induced by pregnancy.  Describe the components of weight gain in pregnancy.  Describe the factors that are implicated in the onset of labour.  Discuss findings on general examination (heart, lungs and breast) of the healthy pregnant woman. Skills objectives  The student should be able to identify the normal changes in the ECG in pregnancy. Attitudinal objectives  The student should be able to explain how s/he would describe to a pregnant woman the reasons why it can be normal to experience the following in pregnancy: a. Breathlessness. b. Urinary frequency. c. Constipation. d. Heat intolerance. Description of teaching session and methods to be used  Prior preparation by answering the self‐assessment questions.  Discussion of the questions.  Comparison of ECG from a non‐pregnant and pregnant woman.  Role‐play of reassuring women about the normal symptoms of pregnancy. 40 Self assessment questions 1. What are the principal functions of the placenta? 2. What are the main effects in pregnancy of: a. Oestrogens? b. Progesterone? 3. What are the changes that occur in the uterine body and cervix from the first to third trimesters of pregnancy? 4. Describe the symptoms that occur in pregnancy that are caused by normal maternal physiological changes in the following systems: a. Cardiovascular. b. Respiratory. c. Gastrointestinal. d. Urological. 5. What are the normal changes in the ECG that can be induced by pregnancy? 6. What is adequate weight gain in pregnancy ? What are the main components of this increase? 7. Which factors are implicated in the onset of labour ? 8. What are the differences between the findings on general examination (heart, lungs and breast) of the healthy pregnant and non‐pregnant woman? Links with matrices 115 Normal pregnancy 41 OBS 02: THE BOOKING VISIT AND NORMAL ANTENATAL CARE Knowledge objectives By the end of a teaching session the student should be able to:  List the haematological investigations performed at the booking visit.  List the infections screened for at the booking visit, and explain the possible adverse sequelae associated with these infections.  Explain the purpose of a first trimester ultrasound scan.  Describe the systems examined in the fetus during an anomaly scan.  Describe a normal antenatal schedule, and the routine investigations performed.  Describe the investigations performed at a routine antenatal visit.  Define position, lie and station of the presenting part as examined in the antenatal clinic.  Describe the psychological preparation performed in the antenatal period before birth. Skills objectives The student should be able to:  Perform a symphysio‐fundal (SFH) height measurement.  Identify the position, lie and station of the presenting part.  Take a blood pressure and perform a urinalysis. Attitudinal objectives The student should be able to explain how s/he would:  Describe to a woman why she needs to attend for regular antenatal checks, even though she is completely healthy.  Explain to a woman the significance of having a reduced SFH measurement and the subsequent management. Description of teaching session and methods to be used  Prior preparation by answering the self‐assessment questions.  Discussion of the questions.  Discuss examples of scans and growth curves in the antenatal period. 42 Self assessment questions 1. Compare and contrast the information that can be gained from a (1) 12 week and (2) 20 week ultrasound scan and its limitations. 2. List the blood tests that are routinely performed at the booking visit and briefly explain for each the effect on antenatal care of a positive and negative result. 3. Discuss the statement "A move to less routine antenatal visits will not affect the standard of care". 4. What are the reasons for antenatal education? 5. A 25 year old black woman books with her midwife at 8 weeks gestation in her first pregnancy. What blood tests should you perform that will be helpful in her antenatal care? 6. The next week she returns to discuss her results. a. She is negative for Rubella antibodies. What advice do you give her? b. She is Rhesus negative. What additional management will be required? c. What dietary advice should she receive? Links with matrices 115 Normal pregnancy 43 OBS 03: PREPREGNANCY COUNSELLING Knowledge objectives By the end of a teaching session the student should be able to:  List the topics that may be discussed in pre‐pregnancy counselling of the apparently healthy woman.  List the screening tests that may be performed pre‐pregnancy in the apparently healthy woman.  Identify maternal conditions that require referral for pre‐pregnancy counselling.  Describe the effects of smoking during pregnancy on the mother and fetus.  Describe the effects of excess alcohol consumption during pregnancy on the mother and fetus.  Discuss the policy of pre‐pregnancy counselling of all apparently healthy woman. Attitudinal objectives  Describe how you would explain the benefits of pre‐pregnancy screening tests to an apparently healthy woman who was uncertain about their usefulness.  Describe the role of the practice nurse in pre‐pregnancy counselling.  Summarise the ethical issues relevant to pre‐pregnancy screening for genetic disorder. Description of teaching session and methods to be used  Prior preparation by answering the self‐assessment questions.  Discussion of the questions.  Role play of a discussion regarding smoking in pregnancy. Self assessment questions 1. What are the topics that should be discussed in pre‐pregnancy counselling of the apparently healthy woman? 2. What are the screening tests that may be performed pre‐pregnancy in the apparently healthy woman? 3. List 10 maternal conditions that require referral to a specialist for pre‐pregnancy counselling and for each briefly explain the reason why. 4. What are the effects of a) smoking and b) excess alcohol consumption during pregnancy on the mother and fetus. What are the recommended minimum intakes for each? 5. Discuss the statement "The policy of pre‐pregnancy counselling of all apparently healthy women is a waste of resources". Links with matrices 115 Normal pregnancy 11 (B) A family history of genetic disorders 44 OBS 04: ANAEMIA IN PREGNANCY Knowledge objectives By the end of a teaching session the student should be able to:  Explain the changes that occur in the haemoglobin concentration, haematocrit, red cell mass, plasma volume, iron metabolism, white cell count and platelet count during normal pregnancy.  List the commonest causes of anaemia in pregnancy in women from the UK in approximate order of prevalence.  List the predisposing factors for iron deficiency anaemia.  Describe the abnormalities found in the MCV, MCH and TIBC for the following causes of anaemia: o Iron deficiency anaemia. o B12 or folate deficiency.  Describe the potential maternal and fetal effects of: o Iron deficiency anaemia. o Sickle cell disease.  Describe the genetic basis for sickle cell disease and the thalassaemias.  Describe the principles of treatment of iron deficiency anaemia in pregnancy.  Describe the problems with oral iron supplements and whys of improving the side‐effect profile.  Evaluate the use of a policy of universal screening for haemoglobinopathy in a) an inner city and b) a suburban population.  Understand the difference between immune thrombocytopaenia and gestational thrombocytopaenia.  Describe the pathophysiology of rhesus disease and the mechanism by which anti‐D prevents this. Skills objectives  The student should be able to interpret the patterns of abnormality found in a full blood count that are indicative of: o Iron deficiency anaemia. o Alpha thalassaemia. o B12 or folate deficiency. Attitudinal objectives  Explain the role of the midwife in the management of anaemia in pregnancy.  Describe the ethical issues relevant to screening for sickle cell disease. Description of teaching session and methods to be used  Prior preparation by answering the self‐assessment questions.  Discussion of the questions.  Role‐play of counselling couple who both have sickle cell trait. 45 Self assessment questions 1. What are the changes that occur in the haemoglobin concentration, haematocrit, red cell mass, plasma volume and iron metabolism during pregnancy? 2. A pregnant woman has a booking FBC that shows a Hb of 9 g/dl, a low MCV, MCH and TIBC. Hb electrophoresis is normal. a. What is the most likely cause of her anaemia? b. How should this be initially treated? c. What are the side effects of this treatment? d. How might you reduce the side‐effects of this treatment? 3. If the Hb is found to be below 7 g/dl at 34 weeks gestation what other treatments can be used? Compare and contrast are the risks and benefits of each of these treatments. 4. What are the most common causes of megaloblastic anaemia in pregnancy and how may they be treated? 5. Describe the genetic basis for sickle cell disease and the thalassaemias. 6. What screening tests for anaemia, haemoglobinopathy and antibodies are performed at the booking visit in a) all women in the UK and b) in women in inner city areas, and why? 7. Compare and contrast the effects of a) iron deficiency anaemia and b) sickle cell disease, on the mother and fetus. Links with matrices 115 Normal pregnancy 5 Anaemia, including haemoglobinopathies 46 OBS 05: THE ANOMALY SCAN AND PRENATAL DIAGNOSIS Knowledge objectives By the end of a teaching session the student should be able to:  List the aims of the 12 week and 20 week anomaly scans.  List 5 major abnormalities that can be detected by an anomaly scan.  List the screening tests that are available for Down's syndrome.  List the two major genetic disorders for which prenatal diagnosis is available.  List the invasive procedures that are used for prenatal diagnosis and the gestations at which they are commonly carried out.  List the complications of invasive diagnostic procedures used in prenatal diagnosis.  Explain the difference between screening for and diagnosis of Down's syndrome.  Describe how you would explain the options available to a woman who is seeking screening for Down's syndrome.  Explain the important factors in deciding the optimal gestation for an anomaly scan.  Describe how you would explain the benefits and drawbacks of an anomaly scan to a woman who is unsure whether to have the procedure.  Explain the influence of religious, cultural or ethical beliefs on decisions related to prenantal diagnosis including Down’s syndrome screening and / or the anomaly scan.  Explain the influence of informed consent prior to prenatal screening tests including the anomaly scan. Skills objectives The student should have observed and be able to describe the following procedures;  Amniocentesis or CVS.  A 1st trimester scan including nuchal translucency.  An anomaly scan.  The student should be able to interpret the following investigations: o The results of Down's syndrome screening tests. o The report from an anomaly scan. Attitudinal objectives  The student should be able to explain the reasons for being sensitive to the ways in which a woman's cultural and religious beliefs affect her perception of prenatal screening and diagnosis. Description of teaching session and methods to be used  Prior preparation by answering the self‐assessment questions.  Discussion of the questions.  Lecture with slides illustrating anomaly scan ± video of anomaly scan and other procedures.  Small group discussion of cases. 47 Self assessment questions 1. A 36 year old woman presents to you, her GP, at 10 weeks gestation. She wishes to discuss the risk that she is carrying a fetus with Down's syndrome and what options are available for testing. a. What is her age‐related risk that the fetus has Down's syndrome? b. What screening tests for Down's would be available to her? 2. She opts to have a screening test and returns to your surgery at 12 weeks with a result showing her risk is now 1 in 110 and asks your advice on how to proceed. a. How would you explain to her what a risk of 1 in 110 means? b. What diagnostic test would be available to her and what are the risks of each? c. How might her religious, cultural and ethical beliefs shape her decision as to how to proceed? 3. A 28 year old lady of African origin comes to see you requesting booking for obstetric care in her first pregnancy. She is now 11 weeks' gestation. a. Is there a particular genetic disorder that you should discuss with this woman? b. What test would you offer to the woman and her partner? 4. The couple undergo testing and the results show that both the woman and her partner carry the disorder. a. How would you explain the risk to the fetus? b. Would prenatal diagnosis be available to the couple and if so what risks would this involve? c. What personal factors might influence the couples' decision as to how to proceed? 5. A 32 year old woman comes to see you at 18 weeks gestation. She has had a nuchal translucency scan and serum screening for Down's both of which indicate a risk lower than the maternal age‐related risk. She tells you that her cousin has epilepsy and so she wishes to have an amniocentesis to make sure "everything is all right" with her baby. a. Does this family history significantly increase the risks to this baby? b. Can an anomaly scan or prenatal diagnostic techniques ensure that "everything is all right" with her baby? c. Is prenatal diagnosis of epilepsy possible? d. How would you attempt to reassure this woman? 6. Concerning the fetal anomaly scan: a. What structural features of the fetus are routinely checked? b. How may it be used to date a pregnancy? c. List 3 markers of chromosomal abnormality that may be found at the anomaly scan. What is the significance of these? Links with matrices 115 Normal pregnancy 116 Unwanted pregnancy and termination of pregnancy 11B A family history of genetic disorder 48 0BS 06: MONITORING THE FETUS Knowledge objectives By the end of a teaching session the student should be able to:  Define small for gestational age and list the causes.  Define intrauterine growth restriction and list the causes.  Understand the principles of antenatal surveillance of the fetus (screening, palpation/SFH assessment, ultrasound) and intrapartum surveillance (intermittent auscultation, CTG and fetal blood sampling). Skills objectives The student should be able to:  Describe and interpret a normal cardiotocogram.  Interpret plots on a fetal growth curve.  Discuss the risk of fetal morbidity and mortality in an IUGR fetus with continuing pregnancy, and role of fetal surveillance in monitoring such a fetus. Description of teaching session and methods to be used  Prior preparation by answering the self‐assessment questions.  Discussion of the questions.  Examination of sample cardiotocograms and fetal ultrasound growth charts. Self assessment questions 1. A women presents to her GP at 34 weeks gestation with reduced fetal movements. What is your management? 2. The same woman is seen on the day unit and on examination is found to have a normal CTG, but the symphysis fundal height is only 30cms. What investigations would you perform? 3. This woman subsequently has an ultrasound scan which shows anhydramnios and there are poor views of the fetus. What is the possible diagnosis? 4. It is decided that this woman should be delivered. What factors would determine the method of delivery? Explain why? 5. Before delivery this woman has reduced fetal movements and is found to have a still birth. How would you explain this to her? Links with matrices 115 Normal pregnancy 49 OBS 07: THE MANAGEMENT OF LABOUR AND DELIVERY Knowledge objectives By the end of a teaching session the student should be able to:  Define the latent phase and the first, second and third stages of labour.  List the observations that are recorded in a partogram and explain the reason why.  Define primary and secondary arrest of labour.  Explain the changes in the position of the fetal head that occur during the second stage of labour.  Evaluate augmentation of labour.  Compare and contrast intermittent auscultation and continuous electronic fetal heart monitoring. Skills objectives  The student should be able to interpret a partogram showing primary and secondary arrest of labour.  The student should have observed and be able to describe: o Normal delivery. o Active management of the third stage. Attitudinal objectives By the end of a teaching session the student should be able to:  Describe how s/he would explain to a woman what occurs during and after delivery.  Explain the role of the midwife in antenatal education concerning labour.  Describe the relevant ethical issues surrounding the decision of a healthy pregnant woman to have an elective Caesarean section rather than undergo a trial of vaginal delivery. Description of teaching session and methods to be used  Prior preparation by answering the self‐assessment questions.  Discussion of the questions.  Interpretation of partograms.  Use of pelvis and fetal models to show mechanism of delivery. 50 Self assessment questions 1. How would you define: a. The latent phase and the first, second and third stages of labour? b. Primary and secondary arrest of labour. 2. Define the following: presentation, lie, attitude, denominator, position, station, engagement, bregma. 3. What do you understand by the terms malpresentation and malposition? 4. What criteria is used to determine the bishop's score? 5. List four indications for an instrumental delivery. 6. What are the prerequisites which must be met before undertaking an instrumental delivery? 7. What is the purpose of the partogram ? 8. What is "active" management of the first stage of labour and what are its advantages and disadvantages ? 9. What are the differences between "active" and "physiological" management of the third stage? 10. A healthy woman at 38 weeks gestation requests an elective Caesarean section rather than a trial of normal delivery. What information would you elicit from her, what information would you give her to help her make her decision and how would you respond? 11. A woman asks you what are the advantages and disadvantages of continuous electronic fetal heart over intermittent auscultation of the fetal heart in labour. How do you reply? Links with matrices 115 Normal pregnancy 51 OBS 08: ANTEPARTUM HAEMORRHAGE Knowledge objectives By the end of a teaching session the student should be able to:  Define antepartum haemorrhage (APH), placental abruption and placenta praevia.  List the causes of APH and indicate their relative prevalence.  Draw a diagram showing the position of the placenta in major and minor placenta praevia.  List risk factors for: o Placental abruption. o Placenta praevia.  Compare and contrast the symptoms and signs found in women with vaginal bleeding secondary to placental abruption and placenta praevia.  Describe the management of a woman with painless vaginal spotting after 24 weeks.  Describe the management of a woman with severe placental abruption.  Describe the principles of management of a woman with placenta praevia.  Describe the possible operative complications of Caesarean section for placenta praevia.  Evaluate the use of ultrasound in the management of: o APH. o Women who have painless vaginal spotting after 24 weeks gestation. Skills objectives  The student should be able to insert a large bore intravenous cannula.  The student should be able to interpret a hard copy ultrasound scan image of the pregnant uterus at > 24 weeks and identify site of the placenta. Attitudinal objectives  By the end of a teaching session the student should be able to discuss how s/he would explain to a pregnant woman who has children at home the reason for her staying in hospital following: o Painless vaginal spotting after 24 weeks gestation for unknown cause. o Vaginal bleeding after 34 weeks gestation for placenta praevia.  By the end of a teaching session the student should be able to discuss how s/he would explain to a woman, whose baby has died following a placental abruption, the reasons why this has happened. Description of teaching session and methods to be used  Prior preparation by answering the self‐assessment questions.  Discussion of the questions.  Role play of describing management of APH (as above) to women.  Examine an ultrasound scan image of placenta praevia and placental abruption and discuss use of ultrasound.  Viva voce assessment of ability to prioritise activities during acute assessment of a woman with a large APH. 52 Self assessment questions 1. Define antepartum haemorrhage (APH) and placenta praevia and list 5 of the most important predisposing factors for each. 2. List 5 causes of APH. 3. What are the a) differences and b) similarities in the symptoms and signs found in women with vaginal bleeding secondary to placental abruption and placenta praevia. 4. What is the normal blood volume of a pregnant woman? 5. What is the purpose of performing a Kleihauer test on someone who presents with an APH? 6. Describe your initial management of a woman at 38 weeks gestation who has lost one litre of blood per vaginum secondary to placental abruption. 7. You are about to obtain consent from a woman who is to undergo Caesarean section for placenta praevia. What possible peroperative complications will you explain to her and how? 8. A women is seen at 34 weeks gestation with an ultrasound report stating that her placenta is covering the internal cervical os. This scan was performed as her anomaly scan had revealed a low placenta. She has had no bleeding per vagina. a. What information would you obtain from her? b. What advice would you give her? c. Are there any investigations that should be performed? d. When and how should she be delivered? Give reasons for your choice. e. If she had a previous Caesarean Section, how would your management be altered? 9. A woman is admitted at 32 weeks singleton gestation with a "woody hard uterus". Her Symphysial Fundal Height measures 38 cms. She feels faint. She also complains of p. v. spotting of blood. a. What immediate action should be taken? b. List the maternal systems that can be adversely affected by her likely diagnosis. c. For each system give a possible diagnosis. d. What other events is she at risk of post partum, and how can these be avoided. e. What role does ultrasound play in the diagnosis and management of this presentation? Links with matrices 2 Circulatory collapse 10 Acute abdominal pain 111 Vaginal bleeding in late pregnancy (after 24 weeks) 5 (B) Anaemia including haemoglobinopamies 53 OBS 09: PRETERM BIRTH Knowledge objectives By the end of a teaching session the student should be able to:  Define: o Preterm Birth. o Preterm premature rupture of the membranes. o Latrogenic prematurity. o Perinatal mortality rate.  List the maternal risk factors for pre‐term labour.  Describe the reasons for tocolytic use.  List the known causes of preterm delivery.  List the fetal consequences of preterm delivery.  Describe the strategies to limit these consequences.  Summarise the methods of predicting preterm labour.  List the causes of iatrogenic prematurity.  Describe the drugs (and their mechanism of action) used for tocolysis.  Describe the role of cervical scanning and cerclage in the prevention of prematurity.  Describe how, why and when antenatal steroid therapy should be prescribed. Skills objectives  Given appropriate equipment the student should be able to perform the following tasks and procedures listed below: o Interpret a contraction trace from a CTG.  The student should able to describe the components of the Bishop score.  The student should be able to interpret the following investigations: o C reactive protein, White Blood Cell Count, Fibronectin vaginal swab, Anticardiolipin antibodies. o Urine culture and sensitivity and microscopy. Attitudinal objectives  Describe how s/he would explain to a patient the reasons for administering antenatal steroids.  Analyse the ethical issues relevant to delivery of a baby at the limits of viability and be able to discuss the issues around fetal monitoring and delivery at 24 to 26 weeks gestation. Description of teaching session and methods to be used  Prior preparation by answering the self‐assessment questions.  Discussion of the questions.  Discussion of clinical case involving ethical issues around the limits of viability. 54 Self assessment questions 1. What is the lower limit of viability in modern obstetric/neonatal practice? 2. Define neonatal death rate. 3. Which maternal risk factors for prematurity are amenable to intervention? 4. When should antenatal steroids be prescribed? What are the disadvantages of giving antenatal steroids? 5. What treatments can be used to prolong pregnancy? 6. Under what circumstances would these treatments be used? 7. How can the fetal consequences of preterm birth be limited antenatally? 8. What is the commonest cause of iatrogenic preterm birth, and how can it be prevented? 9. A 32 year old women at 24 weeks gestation is referred to an Obstetrican. She has had three previous labours at 18,22 and 23 weeks gestation (all died.) a. What history would you take from this women? b. What investigations could be done at this gestation? c. How would her antenatal care differ from a women without such a history? d. Is there any intervention proven to prevent a further similar loss? 10. The same women described above presents in threatened preterm labour at 26 weeks gestation. Her cervix is dilated to 2 cm. and the membranes are bulging. a. What could have caused this dilation? b. What investigations should be performed? c. List the therapeutic options. 55 OBS 10: THE THIRD STAGE AND THE PUERPERIUM Knowledge objectives By the end of a teaching session the student should be able to:  Define the 3rd stage, and the puerperium and list the causes of post partum haemorrhage.  Explain the management of post partum haemorrhage.  Describe the physiological changes that prevent haemorrhage during and after the third stage of labour.  Compare and contrast "active" and "physiological" management of the third stage.  Explain the benefits of breast feeding.  List postnatal complications that cause maternal death.  Explain management of post partum pyrexia.  Explain the role of midwives and health visitors in supporting new mothers. Skills objectives  The student should have observed and be able to describe the active management of the 3rd stage.  The student should have observed breast feeding and should be able to explain lactation to a mother. Description of teaching session and methods to be used  Prior preparation by answering the self‐assessment questions.  Discussion of the questions.  Lecture. Self assessment questions 1. Define the 3rd stage, and the puerperium. 2. List the causes of post partum haemorrhage. 3. Five minutes following a normal delivery a woman is bleeding heavily. What would your management be? 4. Ten days later the same woman presents with a temperature of 38.4C. a. What questions would you ask? b. What will you look for on examination? c. What is the differential diagnosis and what is your treatment of choice? 5. Explain the role of midwives and health visitors in supporting new mothers. Links with matrices 115 Normal pregnancy 134 Infant feeding problems 56 OBS 11: HYPERTENSION AND PROTEINURIA IN PREGNANCY Knowledge objectives By the end of a teaching session the student should be able to:  Define: o Pre‐eclampsia. o Asymptomatic bacteruria. o Urinary tract infection. o Chronic hypertension. o Nephrotic syndrome. o Significant proteinuria in pregnancy and list the causes.  List the risk factors for pre‐eclampsia.  List the causes of secondary hypertension.  List the risk factors for essential hypertension.  Summarise the purpose of blood pressure measurement in antenatal care.  List the maternal organ‐systems that can be affected by pre‐eclampsia and describe these effects.  Describe the principles of management of women pre‐eclampsia.  List the drugs used in the management of hypertension / pre‐eclampsia and describe the reasons for their use and their side effects.  Describe the possible fetal consequences of pre‐eclampsia.  List the signs and symptoms of' ‘fulminating" pre‐eclampsia.  Explain what is meant by eclampsia.  Explain white coat/transient hypertension. Skills objectives  Given appropriate equipment the student should be able to perform the following tasks and procedures listed below: o Measure BP using a mercury sphygmomanometer. o Dipstick urinalysis. o Measure symphysio‐fundal height. o Identify oedema.  The student should able to describe the method of obtaining a clean catch urine specimen.  The student should be able to interpret the following investigations: o Impaired tests of renal (urate, electrolyte, creatinine), hepatic (liver transaminases) function and coagulopathy (platelets, clotting studies). Urine culture and sensitivity and microscopy. Attitudinal objectives  Describe how s/he would explain to a patient the answer to the question "What is pre‐
eclampsia and how can it affect me and my baby?"  Analyse the ethical issues relevant to delivery of a baby for maternal versus fetal safety. 57 Description of teaching session and methods to be used  Prior preparation by answering the self‐assessment questions.  Discussion of the questions.  Taking of BP and testing urine.  Discussion of sample blood test results from women with pre¬eclampsia.  British Hypertension Society CD ROM. Self assessment questions 1. What are the differences between pre‐eclampsia and essential hypertension? 2. What is the importance of asymptomatic bacteruria in pregnancy? 3. How would you define significant proteinuria in pregnancy and what are the causes ? 4. How would you define hypertension in pregnancy and what are the causes ? 5. What is the purpose of blood pressure measurement in antenatal care? 6. Which maternal organ‐systems that can be affected by pre‐eclampsia and describe these effects? 7. Which drugs are used in the management of pre‐eclampsia and what are the reasons for their use? 8. How can pre‐eclampsia affect the fetus ? 9. What do you understand by the term "fulminating pre‐eclampsia" and what are the clinical signs and symptoms of this condition? 10. A 32 year old women at 36 weeks gestation is referred to the Obstetric day unit with a blood pressure of 130/92 mrnHg. Her booking blood pressure was 128/86 mrnHg. She has no abnormal symptoms attributable to pre‐eclampsia (headache, epigastric discomfort, nausea/vomiting, blurred vision), but has been passing urine more frequently. On dipstick analysis she has 1+ proteinuria. a. List the features of this history that are of concern? b. What immediate methods on the day unit could be used to confirm the possibility of pre‐eclampsia? c. What fetal investigations should be arranged if diagnosis of pre‐eclampsia likely? d. What are the possible causes of 1+ proteinuria on dipstick urinalysis. e. How can these causes be differentiated? 11. The same woman described above is found to have sustained hypertension on the day unit (average BP 142/96 mrnHg). She is admitted to hospital and the 24 hour urine sample confirms >300 mg proteinuria. a. What other investigations should be performed on the mother? b. What instructions on ward management should be given? c. List three factors which determine the prognosis? 12. This woman develops severe hypertension in the following week while an inpatient. Her baby is normally grown, and has normal liquor volume and Doppler assessment of the umbilical artery. a. Are there any therapeutic measures to reduce risk to mother and fetus? b. Can this condition be cured, if so how? 58 c. This women has an eclamptic fit. Are they any therapeutic measures to prevent recurrent seizures? Links with matrices 2 (B) High blood pressure ‐ includes pre‐eclampsia 10 Proteinuria 59 0BS 12: MEDICAL DISORDERS IN PREGNANCY Knowledge objectives By the end of a teaching session the student should be able to:  Explain the changes that occur in pregnancy for the following: o Liver blood flow and liver function tests. o Renal blood flow and serum urea, electrolytes and creatinine. o The renal threshold for glucose and resistance to the effects of insulin.  Compare and contrast the effects of pregnancy in general on women with cardiac disease, respiratory disease, renal disease and liver disease.  Compare and contrast the effects of a) hypothyroidism and b) hyperthyroidism on the mother and fetus.  Compare and contrast the general effects on pregnancy of cardiac disease, respiratory disease, renal disease and liver disease.  Describe the general effects of epilepsy on pregnancy and the effects of pregnancy on epilepsy. Outline the effects of anti‐epileptic medication on the fetus.  Describe the methods for screening for diabetes in pregnancy and the reasons why different methods are used according to the nature of the population screened.  Describe the principals of the treatment of diabetes in pregnancy.  Compare and contrast the effects on both the fetus and mother of pre‐pregnancy diabetes and gestational diabetes. Attitudinal objectives  The student should be able to explain how they would describe to a pregnant woman: o The reasons for screening for diabetes in pregnancy. o The effects of poorly controlled diabetes on pregnancy. Description of teaching session and methods to be used  Prior preparation by answering the self‐assessment questions.  Discussion of the questions. Self assessment questions 1. Draw up a table that demonstrates: a. The differing effects of pregnancy in general on women with cardiac disease, respiratory disease, renal disease, liver disease and epilepsy. b. The effects of these conditions may have on pregnancy. 2. List the changes that occur in pregnancy in the following parameters: a. Liver blood flow and liver function tests. b. Renal blood flow and serum urea, electrolytes and creatinine. c. The renal threshold for glucose and resistance to the effects of insulin. 3. What are the changes that occur in insulin and glucose homeostasis during pregnancy? 4. List a) the risk factors and b) the methods of screening for diabetes in pregnancy. 5. Describe the effects of uncontrolled diabetes on the mother and fetus. 60 6. Explain the principles of treatment of diabetes in pregnancy. 7. Explain the general principles of the management of epilepsy in pregnancy. Links with matrices 1 Chest pain 4 Acute and chronic wheezing 8 Palpitations 22 Jaundice 34 Acute anuric renal failure 35 Polyuria 36 Haematuria 63 Loss of consciousness 69 Fits and convulsions 115 Normal pregnancy Bl Hyperglycaemia/glycosuria 61 GYNAECOLOGY GYN 01: MENSTRUAL DISORDERS Knowledge objectives By the end of a teaching session the student should be able to:  Define menorrhagia.  List the causes of menorrhagia, intermenstrual bleeding and postcoital bleeding.  Explain the significance of menorrhagia in the general population.  Define dysfunctional uterine bleeding.  Describe the management of menorrhagia, intermenstual bleeding and postcoital bleeding.  Explain the medical treatments used for the treatment of menorrhagia.  Explain the surgical treatments used in the treatment of menorrhagia.  Formulate a flow diagram for the treatment of menorrhagia. Skills objectives  The student should have observed and be able to describe the following procedures: o Hysteroscopy and endometrial sampling (eg. Pipelle).  Perform a pregnancy test (urine). Attitudinal objectives  Explain the role of the practice nurse in the management of women with menorrhagia. Description of teaching session and methods to be used  Prior preparation by answering the self‐assessment questions.  Discussion of the questions.  Discussion of RCOG flow‐charts for treatment of menorrhagia. Self assessment questions 1. What is the definition of menorrhagia? How is this different from the symptoms women present with and why is this important? 2. Devise a classification for, and list the causes of menorrhagia. 3. What are the causes of intermenstual bleeding and postcoital bleeding? 4. What is the social and economic significance of menorrhagia in the general population? 5. Define dysfunctional uterine bleeding. 6. Classify the medical treatments used in the treatment of menorrhagia. 7. List the surgical treatments used in the treatment of menorrhagia. 8. A woman of 35 yr presents to you complaining of heavy periods with clots and flooding for the last 2 years. Her periods are regular. a. What are the important points to elicit in your history? b. What are the important things to try and elicit in your examination? c. What further investigations would you arrange? 62 9. A 45 year old woman who completed her family comes to you complaining of heavy periods for 10 years and says she wishes to have a hysterectomy. What is your management? Links with matrices 113 Disorders of menstruation 114 Dysmenorrhoea and pelvic pain 5 (B) Anaemia, including haemoglobinopathies 63 GYN 02: MISCARRIAGE AND ECTOPIC PREGNANCY Knowledge objectives By the end of a teaching session the student should be able to:  Define miscarriage.  Explain what is meant by threatened, missed, inevitable, incomplete & complete miscarriage.  Explain the risk factors and natural history of early miscarriage.  Explain expectant, medical and surgical treatments of miscarriage.  Compare and contrast the presentation, treatment and investigation of first and second trimester miscarriages.  Define and list the causes of recurrent miscarriage.  Describe the investigations that should be performed on someone with recurrent pregnancy loss.  Define ectopic pregnancy.  List in order of frequency the possible sites of ectopic pregnancy.  List the predisposing factors for ectopic pregnancy.  Explain expectant, medical and surgical treatments for ectopic pregnancy. Skills objectives  Be able to perform a bimanual examination and estimate the size of an early pregnancy (less than 12 weeks).  The student should have observed and be able to describe an evacuation of retained products of conception (ERPC).  The student should be able to interpret a normal and abnormal karyotype. Attitudinal objectives  Describe how you would explain the management of early embryonic demise to a woman who has presented at 8 weeks gestation. Description of teaching session and methods to be used  Prior preparation and self assessment questions.  Discussion of the questions. 64 Self assessment questions 1. A woman presents in early pregnancy at 12 weeks gestation with vaginal bleeding. On examination the cervical os is closed. a. What aspects of history may determine prognosis? b. What investigations should be performed? c. What is the correct management of this condition? 2. This woman presents 5 days later having passed products of conception. a. What are the management options? b. What possible complications could be associated with this diagnosis? c. If she had three consecutive losses like this what investigations could be performed? 3. The woman returns following counselling, investigations and is found to have a trisomy 21 karyotype on the last fetus. Explain to her the significance of this diagnosis and how this will effect future management. Links with matrices 112 Amenorrhoea 113 Vaginal bleeding in early pregnancy 65 GYN 03: ACUTE PELVIC PAIN INCLUDING OVARIAN CYSTS Knowledge objectives By the end of a teaching session the student should be able to:  Understand the anatomy of lower abdominal pain.  Understand the differences in presentation and symptomatology of visceral versus somatic pain.  List the causes of acute onset pelvic pain.  Understand the anatomy of the mature follicle and what makes an ‘ovarian cyst’.  Compare and contrast presentation, and clinical features of 'physiological' cysts, neoplastic benign cysts, and malignant cysts.  Compare and contrast the signs and symptoms of ectopic pregnancy, an ovarian cyst accident (including haemorrhage, rupture and torsion) and first trimester miscarriage.  Construct a flow diagram for the management of ovarian cysts presenting with acute pelvic pain. Skills objectives The student should have observed and be able to describe:  Transvaginal ultrasound features of physiological cysts, neoplastic cysts and those that could be malignant.  Laparoscopy and its role in the management and treatment of conditions causing acute pelvic pain. Attitudinal objectives By the end of a teaching session the student should be able to:  Explain to a patient in whom a cyst has been confirmed on ultrasound, the implications and likely next steps in their management considering the appearances, symptoms and the woman's age.  Describe how s/he would explain the reasons for laparoscopy to a woman with acute pelvic pain.  Describe how s/he would explain the technique and complications of laparoscopy to a woman with acute pelvic pain. Description of teaching session and methods to be used  Prior preparation by answering the self‐assessment questions.  Discussion of the questions.  Role play of explaining the reasons for and the method and complications of laparoscopy. 66 Self assessment questions 1. What are the causes of acute onset pelvic pain? 2. How does the pain relate to the anatomy and the embryological origins of the lower abdominal organs? 3. What is a follicle and how does it differ from an ovarian cyst? 4. Describe the types of ovarian cysts that may occur. How do 'physiological' cysts differ from neoplastic cysts? What factors would tip the balance from a diagnosis of one or the other? 5. What might be the significance of single versus multiloculation in an ovarian cyst? 6. How might the contents of a cyst differ on ultrasound? 7. What features might make you concerned about malignancy? 8. Describe the presentation of different types of cyst accidents. 9. Construct a flow diagram for the management of ovarian cysts presenting with acute pelvic pain. 10. You are asked to see a 36 year old woman who has developed low abdominal pain. She normally has a regular menstrual cycle but has not had a period for 6 weeks. On examination there is a right sided tender adnexal mass. What investigations are indicated and why? 11. A 28 year old woman is found to have a 3cm cyst in one of her ovaries at routine examination and confirmed on ultrasound scanning. What might this be and how would you manage her concern? Links with matrices 10 Acute abdominal pain 110 Vaginal bleeding in early pregnancy 112 Amenorrhoea 67  GYN 04: DYSMENORRHOEA: CHRONIC PELVIC PAIN AND ENDOMETRIOSIS Knowledge objectives By the end of a teaching session the student should be able to:  List the gynaecological and non‐gynaecologcal causes of chronic pelvic pain.  Define primary and secondary dymenorrhoea and describe the differences in the symptomatology and causes of each.  Describe the treatments available for primary dymenorrhoea.  List the investigations that may be undertaken in the management of chronic pelvic pain and explain the reasons for each test.  List the causes of a) deep and b) superficial dyspareunia.  Explain what is meant by the "pelvic pain syndrome" and describe its management.  Define endometriosis and explain the theories of its aetiology.  List the possible signs and symptoms associated with endometriosis.  Describe the a) medical and b) surgical treatments that can be used for endometriosis.  Evaluate a policy of "laparoscopy for all women with pelvic pain". Skills objectives The student should have observed and be able to describe the following procedures:  Laparoscopy.  Transvaginal ultrasound of the pelvis. Attitudinal objectives By the end of a teaching session the student should be able to:  Explain how to describe the reasons for laparoscopy in the managemnt of pelvic pain.  Describe the possible effects of pelvic pain on social and sexual function.  Describe the ethical issues relevant to hysterectomy for very severe endometriosis in a woman aged 25. Description of teaching session and methods to be used  Prior preparation by answering the self‐assessment questions.  Discussion of the questions.  Lecture. 68 Self assessment questions 1. Make a list of the gynaecological and non‐gynaecological causes of chronic pelvic pain. 2. Make a table comparing the differences between primary and secondary dysmenorrhoea. 3. What investigations may be undertaken in the management of chronic pelvic pain? 4. What are the differences in the causes of i) deep and ii) superficial dyspareunia? 5. How would you define endometriosis ? 6. List the theories of the aetiology of endometriosis. 7. A woman of 35 presents to you in the gynaecology outpatient clinic complaining of severe pelvic pain and deep dyspareunia for 5 years. On examination the left adnexa and uterosacral ligaments are tender. a. What are the possible diagnoses? b. What investigations are indicated? c. What might they show? 8. A 26 year old woman with incapacitating pelvic pain has been found to have mild endometriosis at laparoscopy. She wishes to be treated for her pain, does not wish to become pregnant and does not wish any form of surgery. a. What are the medical treatments available? b. What are their side effects? c. How would your treatment differ if she wanted to become pregnant as soon as possible? Links with matrices 11 Chronic abdominal pain 113 Disorders of menstruation 114 Dysmenorrhoea and pelvic pain 69  GYN 05: AMENORRHOEA Knowledge objectives By the end of a teaching session the student should be able to:  Understand the hypothalamic pituitary ovarian end organ axis and be able to show diagrammatically the various feedback mechanisms and the action of the various hormones on the end organs.  Distinguish primary from secondary amenorrhoea.  Describe a scheme for classifying the causes of amenorrhoea based on the primary site of the problem.  Describe the hormonal changes in the menstrual cycle.  List the long‐term consequences of oestrogen deficiency.  List the appropriate investigations for amenorrhoea and devise a scheme to use these results to reach a diagnosis.  Describe the principles of management available for: o Hypothalamic dysfunction. o Pituitary dysfunction. o Ovarian dysfunction. o In women wishing to achieve pregnancy and those who do not. Skills objectives  The student should be able to take a relevant history from a woman with amenorrhoea.  The student should have been observed and be able to describe the following procedures: o Pregnancy testing on urine. o Transvaginal ultrasound of the pelvis. o Hysteroscopy and endometrial biopsy. o Laparoscopy.  The student should be able to interpret a hormonal profile and distinguish a normal from abnormal karyotype. Attitudinal objectives By the end of a teaching session the student should be able to:  Describe the feelings that a woman with amenorrhoea might have.  Explain the possible misunderstandings that a woman may have about her menstrual function and fertility.  Describe the anxieties a woman may have about whether she can conceive and the effect of her condition on a pregnancy.  Describe the ethical issues relevant to the methods available to achieve pregnancy in women with amenorrhoea eg. Oocyte donation / surrogacy. Description of teaching session and methods to be used  Prior preparation by answering the self‐assessment questions and case histories.  Discussion of the questions and case histories. 70 
Seminar will include discussion of: o Hypothalamic pituitary ovarian axis, and what can go wrong (interactive). o Menstrual cycle review. o Causes of amenorrhoea. o Scheme for establishing the diagnosis from the hormone results and karyotype. o Discussion of case histories. Self assessment questions 1. Draw a diagram showing the hormones involved in and the feedback loops of hypothalamic pituitary ovarian & end organ axis. 2. What is the difference between primary and secondary amenorrhoea? 3. List the common causes of primary amenorrhoea and how they present. 4. List the common causes of secondary amenorrhoea in woman a) under 40 and b) over 40. 5. Describe the hormonal changes in the menstrual cycle. 6. What are the long‐term consequences of ovarian failure? 7. What are the appropriate investigations for ovarian failure? 8. Outline the managements available for: a. Hypothalamic dysfunction. b. Pituitary dysfunction. c. Ovarian dysfunction for those that wish to conceive and those that do not. 9. A fashion model of 25 yr presents at the Gynaecology outpatient complaining that she has not had a period for 3 years. She mentions that because of her work she usually restricts her calorie intake to lOOOKcal/day. a. What is the most likely cause for her amenorrhoea? b. What are the likely long term consequences of prolonged amenorrhoea in this woman ? c. How would you manage her problem? 10. A woman of 35 yr presents to you in the clinic complaining of amenorrhoea for 4 years, increasing hirsutism for 2 years, and a 20% weight gain over 5 years. She is mainly worried about her hirsutism. a. What is the likely cause for her symptoms? b. What investigations would you arrange? 11. A woman of 18 presents to the clinic with primary amenorrhoea. She weighs 56kg, height 148cm. She has normal secondary sexual characteristics and is otherwise well. In the past she was diagnosed with a coarctation of the aorta aged 3 months which was corrected when she was 2 years old. She has been discharged from follow up with the cardiac surgeons. a. What is the likely diagnosis? b. What investigations would you perform? Links with matrices 112 Amenorrhoea 113 Disorders of menstruation 118 Subfertility 115 Normal pregnancy 71  GYN 06: PELVIC MASS Knowledge objectives  List the uterine, ovarian, and tubal pathologies that may present as an abdomino‐pelvic mass.  List the other causes of such a mass.  Describe the clinical features that help in differential diagnosis (history and examination) between these pathologies.  Compare and contrast the presenting features of a fibroid uterus, a pregnant uterus and an ovarian cyst.  Compare and contrast the presenting features of ovarian, endometrium and cervical malignancies.  Explain the investigations of a pelvic mass. Skills objectives  The student should have observed and be able to describe the following procedures: o Abdominal ultrasound of a pelvic mass. o A laparotomy of a pelvic mass.  The student should be able to perform a bimanual examination and determine the clinical size and characteristics of an abdominal mass.  The student should be able to interpret the following investigations: o FBC. o Renal and liver profile. o Tumour markers (Ca 125, CEA, Beta HCG, alpha fetoprotein). Attitudinal objectives  Describe how you would explain to a 50 year old woman the reasons for needing a laparotomy for a pelvic mass of uncertain pathology following investigation.  Describe the options for dealing with a large symptomatic fibroid in a 28 year old women who wishes to maintain her fertility. Description of teaching session and methods to be used  Prior preparation by answering the self assessment questions.  Discussion of the questions. 72 Self assessment questions 1. A 30 year old solicitor presents with a pelvic mass of equivalent to the size of a 20 week gestation. She is concerned she has a malignant ovarian cyst. Describe how the following enquiry could help confirm or reassure her concerns: a. To establish the size and nature: i.
How big is the lump? ii.
How quickly is it changing? iii.
Does it move? iv.
Does it vary in size? b. Associated features: i.
When was your last period? ii.
Has there been any irregular bleeding (between periods/after intercourse)? iii.
Are your periods heavy or painful? iv.
Do you ever feel pain? v.
Do you have problems passing urine? c. General history: i.
When was your last smear and was it normal? ii.
What contraception are you using? iii.
Is there any previous or family history of gynaecological problems? iv.
Has any member of your family had breast cancer? Links with matrices 10 Acute abdominal pain 11 Chronic abdominal pain 12 Abdominal mass/ swelling 13 Pelvic mass 115 Normal pregnancy 73  Self assessment questions 1. Define the terms menopause, climacteric and premature ovarian failure. 2. What are the predisposing factors for an early menopause? 3. What are the hormonal and physical changes that occur during the climacteric. 4. List the short and medium‐term symptoms of the climacteric. 5. List the long‐term effects of the menopause. 6. What are the treatments available for: a. Climacteric hot flushes? b. Vaginal dryness? c. Postmenopausal osteoporosis? 7. What are the different routes by which HRT can be delivered? 8. What are the reasons for the large number of different regimens of HRT? 9. Why are progestogens used in HRT? 10. What are the important risk factors for postmenopausal osteoporosis and cardiovascular disease and what effect does HRT have on these conditions? 11. List the risks and benefits of HRT. 12. A woman of 49 presents to you with a history of hot flushes and night sweats for 2 years. She had a hysterectomy 5 years ago. a. What is the likely diagnosis? b. What are the differential diagnoses? c. What treatments are available for this woman's symptoms? d. What would you say to this woman concerning short and long term side effects of these treatments? 13. The same woman is also concerned about osteoporosis. What investigations could you arrange to help you counsel this woman ? 14. A ballet dancer age 25 years has been referred to you because she presented to A&E 6 weeks earlier with a Colles fracture. She has had no periods for 5 years. How would you manage this woman? Links with matrices 1 Chest pain 89 Back pain and sciatica 91 Musculoskeletal deformity 112 Amenorrhoea 6B Osteoporosis 74  GYN 07: SUBFERTILITY Knowledge objectives By the end of a teaching session the student should be able to:  Define subfertility and infertility.  Explain the chances of conception occurring over time in the normally fertile population.  List what physiological events are required in each partner for conception to occur.  Devise a classification for the causes of subfertility.  List the causes of anovulation.  List the cause of impaired spermatogenesis.  List the causes of tubal damage.  Draw a graph of the changes in serum levels of oestrogens, progesterone, LH and FSH during the menstrual cycle.  Identify women at risk of tubal damage.  Describe the change in the ovary with increasing age and how this affects fertility.  Compare and contrast oogenesis with spermatogenesis.  Explain the steps a couple can take to enhance their chances of a pregnancy occurring spontaneously. Skills objectives The student should be able to:  Take a history from a couple presenting with subfertility.  Explain to women when to have hormone tests in a cycle.  Explain to a man how to produce semen sample for analysis.  The student should be able to interpret the following investigations: o Female early follicular phase hormone profile. o Female luteal phase progesterone. o Male semen analysis. Attitudinal objectives The student should be able to explain:  The principles of dealing sensitively and sympathetically with a subfertile couple.  The difficulties some men may have with producing a semen sample for analysis.  The strain that subfertility may put on the relationship of a subfertile couple.  The ethical issues relevant to the lack of NHS funding for subfertility treatment and the decision to treat based on factors such as age/no children etc.  The general principles of treatment of the infertile couple. Description of teaching session and methods to be used  Prior preparation to answering self assessment question.  Discussion of the questions in pairs or small groups with the feedback to larger groups. 75 Self assessment questions 1. What physiological events are needed for conception to occur? 2. What is meant by subfertility and how common is the problem? 3. Under what broad headings would most causes of subfertility fall? 4. A couple attend clinic/surgery and they have been trying for a baby without success and ask for help, what questions would you ask? What investigations would be useful? 5. Devise a classification of the methods of treatment of the subfertile couple based on the cause. Links with matrices 19 Weight loss 53 Loss of libido 54 Impotence 112 Amenorrhoea 113 Disorders of menstruation 118 Subfertility 76 GYN 08: URINARY PROBLEMS Knowledge objectives By the end of a teaching session the student should be able to:  Define and list the symptoms associated with, and understand the pathophysiology and prevalence of: o Urinary incontinence. o Stress incontinence. o Overactive bladder. o Mixed incontinence. o Voiding difficulties.  Explain the difference between stress incontinence and urodynamic stress incontinence and overactive bladder and detrusor overactivity.  Explain the causes of overflow incontinence.  List the predisposing factors that are associated with stress incontinence.  Explain why urodynamic investigations are important in the diagnosis of causes of urinary incontinence and other causes of lower urinary tract dysfunction.  Describe the conservative and surgical methods of treatment of: o Stress Incontinence / urodynamic stress incontinence. o Overactive bladder / Detrusor overactivity. o Voiding difficulties.  Compare and contrast the effectiveness and techniques of vaginal and suprapubic surgical treatment of urodynamic Stress Incontinence. Skills objectives  The student should be able to understand a urodynamic trace.  The student should have observed and be able to describe what is done during urodynamic investigation. Attitudinal objectives  The student should be able to explain the patient impact and taboos relating to urinary incontinence.  Analyse the ethical issues relevant to the performance of surgical treatment of urinary incontinence without prior urodynamic tests. Description of teaching session and methods to be used  Prior preparation by answering the self‐assessment questions.  Discussion of the questions.  Discussion of a urodynamics trace.  Video of urodynamics procedure. 77 Self assessment questions 1. What is the effect of age on the prevalence of the different causes of urinary incontinence / lower urinary tract dysfunction and why? 2. Define: a. Overactive bladder. b. Urodynamic Stress Incontinence. c. Detrusor overactivity. 3. List the symptoms associated with lower urinary tract dysfunction. Which of these are most often associated with: a. Urodynamic Stress Incontinence. b. Detrusor overactivity. c. Voiding difficulty / Overflow incontinence. 4. What are the causes of overflow incontinence? 5. What is the difference between stress incontinence and urodynamic stress incontinence ? 6. What are urodynamic investigations, and why are they important in the diagnosis of causes of urinary incontinence? 7. Describe the methods of treatment of: a. Stress incontinence / Urodynamic Stress Incontinence. b. Overactive bladder / Detrusor overactivity. c. Voiding difficulty / Overflow incontinence. Links with matrices 28 Sinuses, fistulae and stomata 30 Acute retention of urine Urinary frequency and urgency 31 32 Urinary incontinence 33 Pain on micturition (dysuria) 78  GYN 09: UTEROVAGINAL PROLAPSE Knowledge objectives By the end of a teaching session the student should be able to:  Draw a sagittal section through the pelvis identifying the vaginal wall, cervix and surrounding structures.  Define: o Cystocoele. o Rectocoele. o Enterocoele. o Vaginal vault prolapse. o Uterine prolapse.  Classify uterine prolapse according to severity.  Describe a clinical method of measuring the degree of uterine prolapse.  List the factors associated with uterovaginal prolapse.  Describe the symptoms that are associated with: o Cystocoele. o Rectocoele. o Enterocoele. o Vaginal vault prolapse. o Uterine prolapse.  Explain the methods of treatment of uterovaginal prolapse and select factors that are important in the choice of the best treatment for each individual woman. Skills objectives  The student should be able to place a woman in the Sims position.  The student should have observed and be able to describe the use of the Sims' speculum. Description of teaching session and methods to be used  Prior preparation by answering the self‐assessment questions.  Discussion of the questions and case histories. Self assessment questions 1. Draw a sagittal section through the pelvis identifying the vaginal wall, cervix and surrounding structures and indicate the position of the following; cystocoele, rectocoele, enterocoele, vaginal vault prolapse and uterine prolapse. 2. Draw a sagittal section through the pelvis identifying the vaginal wall, cervix and surrounding structures and show the position of the uterine body and cervix in the different degrees of prolapse. 3. What are the factors associated with uterovaginal prolapse and why? 4. Describe the symptoms that are associated with cystocoele, rectocoele, enterocoele, vaginal vault prolapse and uterine prolapse. 5. What are the methods of treatment of uterovaginal prolapse and which factors are important in the choice of the best treatment? 79 Links with core matrices 26 Rectal and vaginal prolapse 80  GYN 10: MENOPAUSE AND OSTEOPOROSIS Knowledge objectives By the end of a teaching session the student should be able to:  Define the terms menopause, climacteric and premature ovarian failure (POF) and list iatrogenic causes of the menopause.  Define the term osteoporosis.  Describe the hormonal and physical changes that occur during the climacteric.  List the short and medium‐term symptoms of the climacteric.  List the long‐term effects of ovarian failure.  List the treatments available for climacteric symptoms and osteoporosis.  Describe the different regimens of hormone replacement therapy (HRT).  Explain the reasons why progestogens are used in postmenopausal women who have a uterus.  List the important risk factors for osteoporosis and cardiovascular disease.  List the risks and benefits of HRT. Skills objectives  The student should be able to interpret a hormonal profile.  The student should be able to take an adequate history of from a perimenopausal woman and identify risk factors for osteoporosis and cardiovascular disease. Attitudinal objectives  The student should be able to appreciate the effect that hypo‐osetrogenic symptoms and osteoporosis can have on quality of life. Description of teaching session and methods to be used  The student will be expected to read the objectives and self assessment questions prior to the session.  The session will be in the form of an interactive tutorial. 81 GYN 11: FEMALE GENITAL MUTILATION Knowledge objectives By the end of a teaching session the student should be able to:  List reasons given for performing FGM.  List countries where FGM is practiced.  Draw the three main types of FGM and explain what part of the female anatomy is removed. Attitudinal objectives  Describe how you would identify a woman with FGM, and explain to a woman with type 3 circumcision the significance of having a reversal done.  Summarise the law, child protection issues and cultural sensitive issues relevant to FGM. Description of teaching session and methods to be used  Prior preparation by answering the self‐assessment questions.  Discussion of the questions. Self assessment questions 1. What are the reasons given to explain why FGM is performed in some countries? 2. In which countries is FGM performed? 3. Draw a diagram of the three types of FGM and label the anatomy that is involved. 4. Explain the effects of FGM on childbirth. 5. Explain the law in the UK governing FGM. 82  GYN 12: GYNAECOLOGICAL ONCOLOGY Knowledge objectives By the end of the teaching session the student should be able to:  Describe the important features in the epidemiology of gynaecological malignancies.  Discuss the pros and cons of screening for gynaecological cancers with particular reference to the national cervical screening program.  List the different risk increasing and/or reducing factors for different gynaecological cancers.  Understand the value of diagnosing cancer in the pre‐invasive or early stage.  Explain the important presenting features pertaining to each cancer type.  Compare the different pathways taken to investigate a potential gynaecological cancer as well as principles of staging.  Explain main treatment modalities for different gynaecological cancers and understand the value of the multidisciplinary approach to management. Skills Objectives  The student should have observed and be able to describe the following procedures: o Abdominal ultrasound for a pelvic mass/uterine pathology. o EUA and diagnostic hysteroscopy. o Laparotomy and laparoscopy for gynaecological cancer.  The student should be able to perform abdominal and bimanual examination and be able to describe abnormal clinical findings.  The student should be able to interpret the following investigations: o FBC. o Renal and liver profile. o Tumour markers. Attitudinal objectives  Be able to discuss the impact of the cancer diagnosis on the psychosocial wellbeing of the patient.  Describe how you would approach a patient and her family to break the news of cancer diagnosis.  Describe issues surrounding the care of the terminally ill patient. Description of teaching session and methods to be used  Prior preparation by answering the self assessment questions.  Discussion of the questions.  Lecture.  Role play highlighting communication skills and breaking bad news. 83 Self assessment questions 1. List the important presenting symptoms in different gynaecological malignancies. 2. A 47 year old woman with persistent postcoital and intermenstrual bleeding for 3 months; her last cervical smear was over 7 years ago? Describe how you would proceed with history taking and how would you investigate her problem? 3. Why do women in the UK still get cervical cancer despite the national cervical cancer screening program? 4. A 58 year old woman referred by her GP with ascites and pelvic mass. What is your differential diagnosis and investigation pathway? 5. What risk factors are associated with ovarian cancer? 6. A 62 year old woman presenting with vaginal bleeding. She is not on any HRT and her last smear was 2 years previously and was normal. Outline your initial management plan. Links with matrices 13 Pelvic mass 12 Abdominal mass/swelling 11 Chronic abdominal pain 19 Weight loss 113 Disorders of menstruation 84 RAPE AND SEXUAL ASSAULT This seminar will be delivered during the GUM week Aims  An understanding of the prevalence and presentations of sexual assault and its impact on women and men.  An understanding of the health care needs of those who have been sexually assaulted. Knowledge objectives By the end of the teaching session the student should be able to:  Define rape.  Define sexual assault.  Summarise the epidemiology of rape.  Explain the potential consequences of rape and sexual assault for society.  Describe the potential consequences of rape for the individual.  Explain the role of forensic examination, including the relationship between doctor and complainant, and consent.  Summarise Locard's principle.  Describe how DNA technology is used in the investigation and prosecution of crime, and how the risk of DNA contamination can be minimised.  Describe the optimal management of someone who has been sexually assaulted, including: o First aid. o Forensic examination. o Prevention and management of sexually transmitted infections. o Post‐coital contraception. o Psychosocial care. o The Havens (http://www.thehavens.co.uk/). Skills objectives  The student should be able to provide an interpretation of injuries drawn on a body diagram. Attitudinal objectives  By the end of a teaching session the student should be able to explain how s/he would discuss police involvement and forensic examination with someone who says that they have just been raped. Description of teaching session and methods to be used  Interactive talk with Powerpoint / overheads.  Small group discussion about: o Major needs of complainant of rape. o Interpretation of body diagrams. 85 Self assessment questions 1. Compare and contrast the definition of a) rape and b) sexual assault. 2. List the important epidemiological risk factors for rape. 3. Explain the potential consequences of rape and sexual assault for society. 4. Describe the potential consequences of rape for the individual. 5. Explain the purpose of forensic examination, including the relationship between doctor and complainant, and consent. 6. Summarise Locard's principle. 7. Describe how DNA technology is used in the investigation and prosecution of crime, and explain how the risk of DNA contamination can be minimised. 8. Describe the optimal management of someone who has been sexually assaulted, including: a. First aid. b. Forensic examination. c. Prevention and management of sexually transmitted infections. d. Post‐coital contraception. e. Psychosocial care. 86 CONTRACEPTION: Community Reproductive and Sexual Health (CRaSH) You will be allocated to a CRaSH practice OSCE Session. Attendance is compulsory. The session is designed to help you feel prepared for the Summer OSCE exam by becoming confident in your:  Knowledge of the range of all major methods of contraception available.  Ability to assess people and help them make an appropriate informed choice.  Ability to provide people with all relevant information.  Ability to explain advantages and disadvantages of each method.  Skill to identify medical history and therapies that might interact with contraception and put women at risk from pregnancy or serious medical problems.  Skill to assess sexual health needs in a non‐judgemental manner.  Skill to promote positive sexual health. All students, including those with cultural or religious objections to some or all methods of contraception or termination of pregnancy, will be expected to be able to demonstrate that they have the knowledge and skills to conduct consultations as outlined above. This will be tested at the OSCE assessment. The session is run on a small group work basis. Please arrive promptly and register by 09.00am. You will be allocated to a group on arrival. All students will be expected to have read the handouts on the Virtual Campus before arriving at the session. These contain the essential knowledge. Please use these handouts as your reference. Further information may be obtained from the Faculty of Sexual and Reproductive Health www.fsrh.org . Text books rapidly become out of date and may cause confusion. Students will work through a series of question sheets and scenarios that will highlight important points about each major group of contraceptives. There will be an opportunity for feedback and discussion with each tutor. It is difficult to obtain real benefit from the session without having read the handouts. The 4 sessions will cover:  Oral methods, patches and rings.  Barrier methods and Emergency Contraception.  Intrauterine methods – IUD and IUS.  Injectable methods and Subdermal Implants. We will help you to clarify information and translate it into everyday clinical situations. Time is very short and allows us to cover only the major points. We are keen to receive your evaluation so we can continue to improve the session. We hope you really enjoy the session and look forward to meeting you there. 87 Self‐directed learning advice (reading list) 
































2nd Chelsea and Westminster Hospital AIDS care handbook, Gazzard B. A Colour Atlas of Clinical Gynaecology, Tindall VR. Wolfe Medical Publications Ltd 1981. ABC of AIDS (ABC series), Ed Michael W. Adler BMJ publications. ABC of breast diseases 3rd ed, Dixon JM. Oxford Blackwell 2006. ABC of Subfertility, Braude and Taylor. BMJ Books 2004. Atlas of breast examination, Fentiman IS, Hamed H. BMJ Books 1997. Basic Urogynaecology, Cardozo L, Cutner A, Wise B. Oxford University Press 1993. Chronic pelvic pain, initial management, Apr 2005 RCOG Green‐top Guideline 41. Churchill's pocketbook of Obstetrics and Gynaecology, Magowan B. Churchill Livingstone 1997. Clinical Obstetrics and Gynaecology 2nd ed, Magowen B, Owen P, Drife J. Saunders 2009. Creasy and Resnik’s Maternal‐fetal medicine 6th ed. Saunders Elsevier 2008. Early Pregnancy Loss, Management, Oct 2006 RCOG Green‐top Guideline 25. Endometriosis, investigation and management, Oct 2006. RCOG Green‐top Guideline 24. Essential Obstetrics and Gynaecology, Symonds EM, Symonds IM. Churchill Livingstone 2003. Essentials of Obstetrics, Arulkumaran S, Sivanesaratnam V, Chatterjee A, Kumar P. Anshan Publishers 2005. Essentials of Gynaecology, Arulkumaran S, Sivanesaratnam V, Chatterjee A, Kumar P. Anshan Publishers 2005. Essential Reproduction (essentials) 6th ed, Johnson M. Blackwell 2007. Female Genital Mutilation management, Jun 2009. RCOG Green‐top Guideline 53. Female Genital Mutilation, Mohoh C. Radcliffe Medical Press. Fundamentals of Obstetrics and Gynaecology, Oats J, Abraham S. Mosby 2010. Gynaecology by Ten Teachers 19th ed, Monga A and Dobbs S. Hodder Arnold 2011. Gynaecology in Focus, Rymer J, Fish A. Churchill Livingstone 2004. Handbook of diseases of the breast 2nd ed, Dixon JM, Sainsbury RC. London Churchill 1998. Hypertension in pregnancy. The management of hypertensive disorders in pregnancy. NICE Clinical Guideline no.107. http://www.nice.org.uk/nicemedia/live/13098/50418/50418.pdf Johns Hopkins Medicine ‐ Point of Care (IT guide). http://www.hopkinsguides.com/hopkins/ub/index/Johns_Hopkins_HIV_Guide/All_Topics/A Lecture notes in Obstetrics and Gynaecology, Hamilton‐Fairley D. Blackwell and Wiley Publishing 2008. Management of the Menopause ,Rees M, Stevenson J, Hope S, Rozenberg S, and Palacios S. RSM Press 2009. Management of tubal pregnancy, May 2004. RCOG Green‐top Guideline 21. Maternal Medicine; medical problems in pregnancy, Greer, Walters, Nelson Piercy. Elsevier Health sciences. Mims' Medical Microbiology, Goering R, Dockrell H et al. Mosby 2008. Multi‐Agency practice guidelines: Female Genital Mutilation – www.fco.gov.uk/fgm. NICE antenatal & intrapartum guidelines. Obstetrics & the Newborn, Beischer & McKay. Bailliere‐Tindall. 88 
























Obstetrics and Gynaecology, Llewellyn‐Jones D. Mosby 1994. Obstetrics and Gynaecology, Impey L, Child T. Wiley‐Blackwell 2008. Obstetrics and Gynaecology, Pitkin J, Peattie AB, & Magowan BA. Churchill Livingstone 2003. Obstetrics by Ten Teachers, Campbell and Lees. Arnold 2000. Obstetrics Illustrated, Hanretty KP. Churchill Livingstone 2009. Obstetrics in Focus, James D, McEwan A. Churchill Livingstone 2004. Obstetrics. A practical manual, Neuberg R. Oxford Medical Publication. Ovarian cysts, Mar 2009. RCOG Green‐top Guideline 32. Pelvic inflammatory disease, Mar 2009. RCOG Green‐top Guideline 32. Placenta praevia and placenta praevia accrreta: Diagnosis and management, Oct 2005. RCOG Greentop Guideline 27. Pocket essentials of Obstetrics & Gynaecology, O'Reilly B, Bottomley C, Rymer J. Elsevier Health Sciences 2005. Polycystic Ovary Syndrome 2004, Fraser and Kovacs; Clinical Obstetrics and Gynaecology Vol 18,No 5. Postpartum haemorrhage, prevention and management, Jun 2009. RCOG Green‐top Guideline 52. Preparation and revision for the DRCOG, Rymer J, Davis G, Rodin A, Chapman C. Elsevier Health Sciences 2003. Preterm prelabour rupture of membranes, Nov 2006. RCOG Green‐top Guideline 44. RCOG Green‐top guidelines www.rcog.org.uk. Recurrent miscarriage, investigation and treatment of couples, May 2003. RCOG Green‐top Guideline 17. Severe pre‐eclampsia/eclampsia, management, Mar 2006. RCOG Greentop Guideline 10A. Sexually Transmitted Diseases 4th ed, Holmes et al. McGraw 2008. Symptoms and Signs in Gynaecology, Amais AG. Found in: Gynaecology, Shaw R, Soutter P and Stanton S. Churchill Livingston 1992. The Handbook of Obstetric Medicine 3rd ed, Nelson‐Piercy C. Informa Healthcare 2006. Training in Obstetrics & Gynaecology:the essential curriculum, Sarris I, Bewley S, Agnihotri S. Oxford University Press 2009. UK National Guidelines for HIV Testing 2008, prepared jointly by British HIV Association, British Association of Sexual Health and HIV, British Infection Society (www.bhiva.org). Urodynamic stress incontinence, surgical treatment, Oct 2003. RCOG Green‐top Guideline 35. Vaginal vault prolapse, Oct 2007. RCOG Green‐top Guideline 46. 89 REPRODUCTIVE AND SEXUAL HEALTH SYMPOSIA GUIDELINES/ LEARNING OBJECTIVES Clinical Aspects of Breast Diseases • To teach students to take an appropriate history from a patient presenting with a breast lump and other breast problems. • To ensure that students understand the concerns/fears that a patient may have when presenting with a breast lump or other breast problems. • To teach students to carry out a clinical examination of the breast. • To demonstrate and show the importance of the triple approach to breast diagnosis. • Over view of breast pathology both benign and malignant. • Overview of breast radiology in symptomatic patients. • To briefly outline management of breast problems. • NB Details of management of breast cancer and breast screening are discussed in the seminars. Abnormal Cervical Smear • List the pros and cons of cervical screening. • Explain the objectives of the National Cervical Screening Programme. • Describe the cervical transformation zone and explain its significance. • Define dyskaryosis and explain its significance. • Describe patient attitudes to cervical screening, their response to both a normal and abnormal smear results and discuss informed consent in relation to this test. • Explain the importance of HPV in cervical pathology. • Explain the importance of colposcopy to detect and treat cervical pathology. • Describe the health promotion opportunities in a cervical screening consultation. Pelvic pain • To be able to give a differential diagnosis of the causes of pelvic pain. • To understand the clinical presentation in terms of history and examination of the common causes of pelvic pain. • To be able to describe the pathological processes involved in the aetiology of the common causes of pelvic pain. • To understand the radiological and laboratory investigations used in the diagnosis of causes of pelvic pain. • To understand the treatment modalities including drug therapy used in patients presenting with pelvic conditions. • To understand the issues caused by the sequelae of pelvic conditions in women. Pregnancy in Perspective • To understand the principles of risk assessment in pregnancy. • To understand the principles and provision of antenatal care. • To understand principles of intrapartum fetal monitoring. • To understand principles of management of labour. • To understand the evidence base in intrapartum care. 90 Termination of pregnancy • Describe the techniques of termination of pregnancy according to duration of pregnancy. • Describe (briefly) the epidemiology of termination of pregnancy in the UK. • Explain the law in relation to the termination of pregnancy in the UK. • Describe the patients perspective while they go through termination. • Describe the role of contraceptive services in the prevention of unplanned pregnancy. • Give a practical idea of consultation with a patient requesting termination and recognition of underlying problems if there are any. • By the end of the symposium students should recognise the importance of the provision of accurate information in a non‐judgemental way to patients requesting termination of pregnancy. Prescribing in pregnancy and while breastfeeding • Principles of prescribing in pregnancy and lactation. • Risk benefit ratios for medications. • Therapeutics and common medical conditions in pregnancy and during lactation. • Substance misuse and pregnancy. • Management of thromboembolic disease in pregnancy and puerperium. Risk management • Introduction for undergraduates to the concepts of risk management in general, including good note‐keeping, the use of agreed protocols, informed consent, preventative action in high‐risk cases, and mechanisms for dealing with difficult cases and complaints. The aim is for students to appreciate the skills relating to risk management in order to be informed rather than apprehensive about risk. Genetics • Tests available. • Who should be offered the tests? • Is screening justified for all pregnant women? • What counselling should be given? • What is the accuracy of the tests? • What happens when a different abnormality is detected from the one tested for? • Prenatal treatment. 91 Ethical dilemmas • To make students aware of normative dimensions of clinical decisions, so that (a) they are able to identify which aspects of decisions are technical in nature and what are ethical and (b) they are able to assess how technical and ethical aspects relate to each other. • To develop and acquire skills in analysing the normative dimension of clinical decisions (identifying moral principles and rules; critically analysing moral arguments). • To develop and acquire skills of exploring and justifying personal decisions regarding ethical issues as they arise in specific clinical contexts. • The symposium will enable students to work on the interface between clinical topics and ethical issues. Gynaecological cancers • Overall, to consider the presentation and management of the most important gynaecological cancers ‐ namely, invasive cervical squamous carcinoma, endometrial adenocarcinoma, and surface epithelial tumours of the ovary. • To understand their basic clinical pathology, including the principles of tumour staging. • To appreciate the role of imaging (MRI and other modalities) in the pre‐treatment staging of gynaecological cancer and in follow up. • To understand the relative roles of surgery, radiotherapy and chemotherapy in the management of these cancers. • To talk to and learn from the experiences of cancer patients under current therapy. 92 SECTION 3 SKILLS ASSESSMENT CHECKLISTS The checklists below correspond to the marking scheme or guidelines for marking that will be used by assessors when evaluating/ signing up your skills during the block. Take a comprehensive obstetric history  Able to calculate the estimated date of delivery from the last menstrual period (LMP).  Present obstetric history and demonstrate ability to: o Obtain relevant past obstetric history. o Present current antenatal course in understandable and logical fashion. Examine a pregnant woman  Friendly and empathic relationship with woman Correct measurement of blood pressure. Correct dipstick urinalysis.  Positions woman to ensure: o Comfort. o Adequate exposure.  Correct measurement of symphysio‐fundal height.  On woman in 3rd trimester (singleton fetus), demonstrate ability to determine: o Lie. o Engagement. o Presentation.  Demonstrate ability to equate clinical findings to clinical significance (e.g. correct size for dates). Insert a cusco speculum and take an endocervical swab and cervical smear  Appropriate introduction.  Explains procedure.  Before examination; o Assembles speculum. o Labels pot.  Parts labia and inserts speculum gently.  Identifies cervix .  Takes smear correctly.  Puts sample into pot appropriately.  Takes endocervical swab correctly.  Removes speculum gently. Bimanual examination  Abdominal examination before pelvic examination.  Parts labia with fingers of left hand. 93  Inserts two fingers of the right hand gently into vagina.  Left hand appropriately placed on abdomen. Assessment of:  Uterus, cervix.  Adnexae. Participate in a normal delivery  To be assessed by senior midwife/ midwifery sister.  Friendly and empathic relationship with: o Labouring woman. o Partner. o Friends and relatives.  Friendly good working relationship with: o Midwife in charge of delivery. o Other staff involved in care.  Willing to help with physical aspects of care of labouring woman.  Present during most of the labour and the all of the delivery.  Showed understanding of progress of labour.  Showed understanding of results of fetal monitoring.  Showed understanding of reasons for all obstetric interventions.  Showed appropriate interest in the newborn.  Willing to help with physical aspects of care of newborn.  Remained with the woman for most of the time following delivery. Catheterisation of a female patient Principles: The female urethra is much shorter than the male urethra and hence infection is more easily introduced into the urinary tract. The introduction of any foreign body into the urethra may introduce infection, thus this procedure must be performed with a no touch or sterile technique. There are two types of catheters: 1. Indwelling which uses a balloon catheter (Foley 5 ‐ 30ml balloon) of various diameters (usually a 12 or 14 French is adequate) and is for continuous drainage of the bladder. 2. Single use sterile for instant drainage of the bladder. Technique: 1. Prepare the items laid out on a sterile pack: a. Foley (balloon) catheter with syringe containing 5‐30mls of sterile water or saline depending on the balloon size or a single use catheter. b. Catheter bag or receptacle for the urine. c. Small gallipot containing antiseptic solution. d. Swabs with which to clean the urethral meatus. e. Sterile gloves. f. Analgesic gel. 94 2. The procedure should be conducted in privacy with a chaperone or helper. 3. Explain to the patient gently what you are about to do and gain their permission and confidence. 4. The patient should be positioned lying on their back with their legs parted with knees bent and feet flat on the bed to reveal the vulva ‐ heels together is more comfortable for some patients. 5. Having donned sterile gloves prepare the trolley by connecting what needs to be, and removing the end of the Foley catheter from the covering plastic bag, fill the syringe etc. 6. The labia minora are parted with one hand and the urethral meatus identified (the clitoris and the vaginal opening should also be identified in order not to try and catherise either of them!). 7. Using the other hand, and keeping the labia parted, the urethral meatus is cleaned with an antiseptic moistened swab. 8. The tip of the catheter (having been moistened with some antiseptic or sterile analgesic gel) is inserted slowly and gently into the urethra to a depth of about 5cms for a Foley or until urine is seen draining. Collect the draining urine into a receptacle or inflate the balloon and connect the urine bag. 9. Clear up your mess disposing of any sharp instruments appropriately. Checklist for assessment of female catheterisation • Prepares the items laid out on a sterile pack. • Explain to the patient gently what is about to be done/ is being done. • Correct positioning. • Correct preparation prior to catheterisation. • Labia minora are parted with one hand correctly. • Urethral meatus identified correctly. • Urethral meatus is cleaned with swab, keeping the labia parted. • Catheter inserted correctly. • Correct collection of urine. • Clean up trolley. Perform a pregnancy test Clearview EASY HCG Clearview EASY HCG is intended for qualitative detection of human chorionic gonadotrophin (hCG) in urine as an aid in the diagnosis of pregnancy. HCG normally begins to be detected in the urine from 7 days after conception. The sudden rapid rise in concentration of hCG in urine following conception makes it an excellent marker for pregnancy. Please read an instruction leaflet carefully – available in all packs. Pregnancy tests are performed widely in gynaecology The best places to observe and then learn to do one independently is the Emergency Gynaecology Unit (EGU), Day Surgery Unit (DSU) and the Surgical Assessment Unit (SALS). 95 Interpretation of Results The test can be read 3‐10 minutes after applying the sample, regardless of which test procedure is used. 





A blue, Control Line (C) must appear within 3 minutes to show that the test has worked correctly. A positive result: two blue lines – a Result Line ® and a Control Line (C) ‐ in the Result Window within 3 minutes. One line may be fainter or darker than the other but this does not affect the interpretation of the result. A negative result: one blue line – a Control Line (C) ‐ in the Result Window within 3 minutes. An invalid result: no blue line in the Control Line (C) region in the Result Window within 3 minutes. If no Control Line appears within 3 minutes, the test is invalid and must be repeated with a new Clearview EASY HCG device. Any result that appears after 10 minutes must be ignored. 96 SINGLE BEST ANSWER AND EXTENDED MATCHING EXAMPLE QUESTIONS SINGLE BEST ANSWER Pain in Pregnancy 1. A previously fit woman of 25 years old at 34 weeks gestation complains to you in the antenatal clinic of mild RIF pain for 10 days. She is mildly tender in this area. What is the most likely cause of her pain? a. Ectopic pregnancy b. Appendicitis c. Volvulus d. Placental abruption e. Irritable bowel syndrome T 2. This woman presents 5 weeks later at 39 weeks gestation complaining of pain at the uterine fundus, fresh vaginal bleeding and is tender over the area of pain. Which one of the following is the most likely diagnosis? a. Ruptured ectopic pregnancy b. Placenta praevia c. Placental abruption T d. Ruptured uterus e. Threatened miscarriage 3. The same woman presents to her GP 8 weeks after a normal delivery complaining of continuous vaginal bleeding, low abdominal pain and occasional fever. What is the most likely cause of her pain and bleeding? a. Endometriosis b. Endometritis T c. Choriocarcinoma d. Retroverted uterus e. Combined contraceptive pill use 97 EXTENDED MATCHING QUESTIONS Theme = Pain and vaginal discharge For each of the questions below choose the single most likely diagnosis from the statements above. Each statement above may be used once, more than once, or not at all. A. Ectopic pregnancy B. Threatened abortion C. Appendicitis D. Twisted ovarian teratoma E. Molar pregnancy F. Pelvic inflammatory disease G. Ruptured bowel diverticulum H. Urinary tract infection I. Renal calculus J. Pyosalpinx K. Twisted hydatid of Morgagni L. Trichomonas vaginalis infection M. Pre‐eclampsia N. Beta haemolytic streptococcal infection O. Lactobaccilus infection 1. A woman of 35 complains of RIF pains 2 weeks after a missed period. She is tender on suprapubic palpation and on vaginal examination the cervical os is closed. Answer = A 2. A 23 year old woman is admitted with pyrexia, lower abdominal pain and purulent vaginal discharge. A pregnancy test is negative and no periods have been missed recently. A pelvic ultrasound scan shows no abnormality. Answer= F 3. A 40 year old woman is seen in A & E with RIF pain, fever, loss of appetite and rebound and guarding in the area of pain. Answer = C 4. A 34 year old woman is seen in the ANC at 26 weeks with dysuria, frequency, fever and proteinuria on dipstick analysis of the urine. Answer = H 5. An otherwise well woman of 17 presents to you complaining of a fishy smelling vaginal discharge and vulval itching. Answer = L 98